Test 3 Practice Questions

अब Quizwiz के साथ अपने होमवर्क और परीक्षाओं को एस करें!

31. The parent of a child hospitalized with acute glomerulonephritis asks the nurse why blood pressure readings are being taken so often. The nurse's BEST reply is: 1. "Blood pressure changes are a common side effect of antibiotic therapy." 2. "Blood pressure changes are a sign that the condition has become chronic." 3. "Acute hypertension, or high blood pressure, must be anticipated and identified." 4. "Hypotension, or low blood pressure, leading to sudden shock can develop at any time."

"Acute hypertension, or high blood pressure, must be anticipated and identified." Blood pressure does not commonly fluctuate with antibiotic therapy. Blood pressure fluctuations do not indicate chronic disease. Most children with glomerulonephritis fully recover. Vital signs, in particular blood pressure, provide information about the severity of the disease and early signs of complications. Acute hypertension is anticipated and requires frequent monitoring for early intervention. Hypertension is more likely with glomerulonephritis.

251. A child is scheduled for a tonsillectomy in the day-stay surgical unit. On the day following surgery, the mother calls the surgical unit and expresses concern because the child has a very bad mouth odor. The nurse makes which response to the mother? 1. "The child probably has an infection." 2. "You need to contact the health care provider immediately." 3. "Bad mouth odor is normal and may be relieved by drinking more liquids." 4. "Have the child gargle with mouthwash."

"Bad mouth odor is normal and may be relieved by drinking more liquids." Rationale: Bad mouth odor is normal following tonsillectomy and may be relieved by drinking more liquids. Options 1, 2, and 4 are incorrect. Additionally, mouthwash gargles will irritate the throat.

301. A nurse is collecting data on a child recently diagnosed with glomerulonephritis. Which of the following questions to the mother would elicit information about the cause of this disease? 1. "Did your child sustain any injuries to the kidney area?" 2. "Did your child recently complain of a sore throat?" 3. "Has your child had any diarrhea?" 4. "Have you noticed any rashes on your child?"

"Did your child recently complain of a sore throat?" Rationale: Group A beta hemolytic streptococcal infection is a cause of glomerulonephritis. Often the child becomes ill with streptococcal infection of the upper respiratory tract and then develops symptoms of acute poststreptococcal glomerulonephritis after an interval of 1 to 2 weeks. The questions to the mother in options 1, 3, and 4 are unrelated to a diagnosis of glomerulonephritis.

299. A nurse is collecting data on an infant with a diagnosis of suspected Hirschsprung's disease. Which of the following questions to the mother will most specifically elicit information regarding this disorder? 1. "Does your infant have foul-smelling, ribbon-like stools?" 2. "Is your infant constantly vomiting?" 3. "Does your infant constantly spit up feedings?" 4. "Does your infant have diarrhea?"

"Does your infant have foul-smelling, ribbon-like stools?" Rationale: Chronic constipation, beginning in the first month of life and resulting in pellet-like or ribbon-like stools that are foul smelling, is a clinical manifestation of Hirschsprung's disease. Delayed passage or absence of meconium stool in the neonatal period is the cardinal sign. Bowel obstruction, especially in the neonatal period, abdominal pain and distention, and failure to thrive are also clinical manifestations. Options 2, 3, and 4 are not specific clinical manifestations of this disorder.

185. The mother of a child with juvenile idiopathic arthritis calls the nurse because the child is experiencing a painful exacerbation of the disease. The mother asks the nurse if the child should perform range-of-motion (ROM) exercises at this time. The nurse makes which response to the mother? 1. "Avoid all exercise during painful periods." 2. "The ROM exercises must be performed every day." 3. "Have the child perform simple isometric exercises during this time." 4. "Administer additional pain medication before performing the ROM exercises."

"Have the child perform simple isometric exercises during this time." Rationale: During painful episodes, hot or cold packs, splinting, and positioning the affected joint in a neutral position help to reduce the pain. Although resting the extremity is appropriate, it is important to begin simple isometric or tensing exercises as soon as the child is able. These exercises do not involve joint movement.

91. Which of the following would cause a nurse to suspect that an infection has developed under a cast? Cold toes Increased respirations Complaint of paresthesia "Hot spots" felt on the cast surface

"Hot spots" felt on the cast surface If hot spots are felt on the cast surface, they usually indicate infection beneath the area. This should be reported so that a window can be made in the cast to observe the site. Cold toes may indicate too tight a cast and need further evaluation. Increased respirations may indicate a respiratory infection or pulmonary emboli. This should be reported, and the child should be evaluated. The five Ps of ischemia from a vascular injury include pain, pallor, pulselessness, paresthesia, and paralysis. Paresthesia is an indication of vascular injury, not infection.

243. A nurse is providing dietary instructions to the mother of a child with celiac disease. Which statement by the mother indicates a need for further instructions? 1. "I can give my child rice." 2. "I am so pleased that I won't have to eliminate oatmeal from my child's diet." 3. "My child loves corn. I will be sure to include corn in the diet." 4. "I will be sure to give my child vitamin supplements every day."

"I am so pleased that I won't have to eliminate oatmeal from my child's diet." Rationale: Dietary management is the mainstay of treatment for the child with celiac disease. All wheat, rye, barley, and oats should be eliminated from the diet and replaced with corn and rice. Vitamin supplements, especially fat-soluble vitamins and folate, may be needed in the early period of treatment to correct deficiencies.

313. A nurse provides instructions to the parents of an infant with hip dysplasia regarding care of the Pavlik harness. Which statement, if made by one of the parents, indicates an understanding of the use of the harness? 1. "I need to remove the harness to feed my infant." 2. "I need to remove the harness to change the diaper." 3. "My infant needs to remain in the harness at all times." 4. "I can remove the harness to bathe my infant."

"I can remove the harness to bathe my infant." Rationale: The harness should be worn 23 hours a day and should be removed only to check the skin and for bathing. The hips and buttocks should be supported carefully when the infant is out of the harness. The harness does not need to be removed for diaper changes or feedings. Option 3 is incorrect.

265. A nurse provides home care instructions to the parents of an infant following surgical intervention for imperforate anus and tells the parents about the procedure for anal dilation. Which statement by the parents indicates the need for further instructions? 1. "I need to use only dilators supplied by the health care provider." 2. "I need to use a water-soluble lubricant." 3. "I will insert the dilator no more than 1 to 2 cm into the anus." 4. "I will insert a glycerin suppository before the dilation."

"I will insert a glycerin suppository before the dilation." Rationale: Following this surgery, anal dilation at home by the parents is necessary to achieve and maintain bowel patency. Inserting a glycerin suppository before dilation is not a component of this procedure. Options 1, 2, and 3 are accurate instructions and will prevent damage to the rectal mucosa.

288. The mother of an infant newly diagnosed with cystic fibrosis is being taught proper nutritional needs for the infant. The nurse determines that the mother understands nutritional needs when the mother replies: 1. "I know that my infant needs to drink predigested formula until she has her stool pattern developed." 2. "When I begin feeding my infant cereal, I will make sure to warm the cereal and administer the pancreatic enzyme mixed in." 3. "I will make sure that I give my infant fat-free milk as a supplement to her predigested formula, because she is not able to digest fat." 4. "I know I need to monitor my infant's stools and if there are more than four stools a day, I will increase the pancreatic enzyme."

"I know I need to monitor my infant's stools and if there are more than four stools a day, I will increase the pancreatic enzyme." Rationale: Cystic fibrosis requires a high-calorie, high-protein diet with pancreatic enzyme replacement therapy. The infant needs to remain on the predigested formula until 1 year of age, when formula can be discontinued and then fat-free milk consumed. The pancreatic enzyme should not be mixed with warmed foods because this inactivates the enzyme. Stools must be monitored, and pancreatic enzymes are administered based on the stool pattern.

190. A nurse provides information to the mother of a 2-week-old infant who was diagnosed with clubfoot at the time of birth. Which statement by the mother indicates the need for further instruction regarding this disorder? 1. "Treatment needs to be started as soon as possible." 2. "I realize my child will require follow-up care until full grown." 3. "I need to bring my child back to the clinic in 1 month for a new cast." 4. "I need to come to the clinic every week with my child for the casting."

"I need to bring my child back to the clinic in 1 month for a new cast." Rationale: The treatment for clubfoot is started as soon as possible after birth. Serial manipulation and casting are performed at least weekly. If sufficient correction is not achieved within 3 to 6 months, surgery is usually indicated. Because clubfoot can recur, all children with the condition require long-term interval follow-up until they reach skeletal maturity to ensure an optimal outcome.

161. A nurse reinforces home-care instructions to the parents of a child with hepatitis regarding the care of the child and the prevention of the transmission of the virus. Which statement by a parent indicates a need for further instruction? 1. "Frequent handwashing is important." 2. "I need to provide a well-balanced, high-fat diet to my child." 3. "I need to clean contaminated household surfaces with bleach." 4. "Diapers should not be changed near any surfaces that are used to prepare food."

"I need to provide a well-balanced, high-fat diet to my child." Rationale: The child with hepatitis should consume a well-balanced, low-fat diet to allow the liver to rest. Options 1, 3, and 4 are components of the home-care instructions to the family of a child with hepatitis.

253. A nurse provides instructions to the mother of an infant with cleft palate regarding feeding. Which statement by the mother indicates a need for further instructions? 1. "I need to use a nipple with a small hole to prevent choking." 2. "I need to stimulate sucking by rubbing the nipple on the lower lip." 3. "I need to allow my infant time to swallow." 4. "I need to allow my infant to rest frequently to provide time for swallowing what has been placed in the mouth."

"I need to use a nipple with a small hole to prevent choking." Rationale: The mother should be taught the ESSR method of feeding the child with a cleft palate: ENLARGE the nipple by cross-cutting a hole so that food is delivered to the back of the throat without sucking; STIMULATE sucking by rubbing the nipple on the lower lip; SWALLOW; then REST to allow the infant to finish swallowing what has been placed in the mouth.

304. A nurse has provided discharge instructions to the mother of an 18-month-old child following surgical repair of hypospadias. Which statement by the mother indicates a need for further instruction? 1. "I should carry my child by straddling the child on my hip." 2. "I should use double diapers to hold the surgery site in place." 3. "I should avoid toilet training right now." 4. "I should encourage fluid intake."

"I should carry my child by straddling the child on my hip." Rationale: Parent teaching following hypospadias repair includes restricting the child from activities that put pressure on the surgical site. Straddling the child on the hip will cause pressure on the surgical site. The parents should be instructed to use double diapers to hold the stent in place and should be instructed how to hold the child during the postoperative period. Fluids should be encouraged to maintain hydration. Toilet training should not be an issue during this stressful period

278. When instructing the caregiver of a child about cast care, the nurse anticipates the need for further teaching when the caregiver states: 1. "I will encourage my child to avoid standing for too long." 2. "I will instruct my child to not put anything inside the cast." 3. "I will allow my child to put cotton balls inside the cast to relieve pressure." 4. "I will encourage my child to keep the injured extremity elevated while resting."

"I will allow my child to put cotton balls inside the cast to relieve pressure." Rationale: Cast care includes keeping the casted extremity elevated on pillows or similar support for the first day, or as directed by the health professional; elevating a lower limb when sitting and avoid standing for too long; encouraging frequent rest for a few days; keeping the injured extremity elevated while resting; and not allowing the child to put anything inside the cast.

184. A nurse is providing instructions to the parents of a child with scoliosis regarding the use of a brace. Which statement by a parent indicates the need for further instruction? 1. "I need to have my child wear a soft fabric under the brace." 2. "I will apply lotion under the brace to prevent skin breakdown." 3. "I need to encourage my child to perform the prescribed exercises." 4. "I need to avoid applying powder under the brace, because it will cake."

"I will apply lotion under the brace to prevent skin breakdown." Rationale: The use of either lotions or powders should be avoided, because they can become sticky or cake under the brace, thus causing irritation. Options 1, 3, and 4 are appropriate statements regarding the care of a child with a brace.

178. A nurse is reinforcing discharge instructions to the mother of a 2-year-old child who has had an orchiopexy to correct cryptorchidism. Which of the following statements, if made by the mother of the child, indicates that further teaching is necessary? 1. "I'll check his temperature." 2. "I'll give him medication so he'll be comfortable." 3. "I'll let him decide when to return to his play activities." 4. "I'll check his voiding to be sure there are no problems."

"I'll let him decide when to return to his play activities." Rationale: All vigorous activities should be restricted for 2 weeks after surgery to promote healing and prevent injury. This will prevent dislodging of the suture, which is internal. Normally 2-year-old children will want to be very active. Therefore, allowing the child to decide when to return to his play activities may prevent healing and cause injury. The parents should be taught to monitor the child's temperature, provide analgesics, as needed, and monitor the urine output.

231. A nurse is evaluating the parent's understanding of discharge care regarding the functioning of the infant's ventricular peritoneal shunt. Which statement by a parent indicates an understanding of the shunt complications? 1. "If my baby has a high-pitched cry, I should call the doctor." 2. "I should position my baby on the side with the shunt when sleeping." 3. "My baby will pass urine more often now that the shunt is in place." 4. "I should call my doctor if my baby refuses purees."

"If my baby has a high-pitched cry, I should call the doctor." Rationale: If the shunt is broken or malfunctioning, the fluid from the ventricle part of the brain will not be diverted to the peritoneal cavity. The cerebrospinal fluid will build up in the cranial area. The result is intracranial pressure, which then causes a high-pitched cry in the infant. The baby should not have pressure when on the shunt side. Skin breakdown and possible compression to the apparatus could result. This type of shunt affects the gastrointestinal system, not the genitourinary system. Option 4 is only a concern if the baby becomes malnourished or dehydrated, which could then raise the body temperature. Otherwise, refusal to eat purees has no direct relationship to the shunt functioning.

201. A nurse is preparing a 2-year-old child with suspected nephrotic syndrome for a renal biopsy to confirm the diagnosis. The mother asks the nurse, "Will my child ever look thin again?" The nurse appropriately responds by saying: 1. "Do you feel guilty because about your child's weight gain?" 2. "In most cases, medication and diet will control fluid retention." 3. "Wearing loose-fitting clothing should help conceal the extra weight." 4. "When children are little, it's expected that they'll look a little chubby."

"In most cases, medication and diet will control fluid retention." Rationale: It is important to give the mother information that addresses the issue that is the parent's concern. Most children experience remission with treatment. Options 1 and 3 are nontherapeutic and may add to the mother's guilt. Option 4 does not acknowledge the concern and is a stereotypical response.

148. Which statement by the parent of a child with polycystic kidney disease and stage 2 renal disease indicates the need for more teaching?

"My child will outgrow this disease." Rationale: this is a lifelong disease

197. A nurse is assisting in collecting data on a child with seizures. The nurse is interviewing the child's parents to establish their adjustment to caring for their child with a chronic illness. Which statement by a parent would indicate a need for further teaching? 1. "Our child sleeps in our bedroom at night." 2. "We worry about injuries when our child has a seizure." 3. "Our child is involved in a swim program with neighbors and friends." 4. "Our babysitter just completed cardiopulmonary resuscitation (CPR) training."

"Our child sleeps in our bedroom at night." Rationale: Parents are especially concerned about seizures that might go undetected at night. The nurse should suggest a baby monitor. Reassurance by the nurse should ensure parental confidence and decrease parental overprotection. Option 2 is a common concern. Options 3 and 4 demonstrate the parents' ability to choose respite care and activities appropriately. The parents need to be reminded that, as the child grows, they cannot always observe their child, but their knowledge of seizure activity and care are appropriate to minimize complications.

310. A child is brought to the emergency department, and a fracture of the left lower arm is suspected. The mother states that the child was rollerblading and attempted to break a fall with an outstretched arm. The child receives diagnostic x-rays, from which it has been determined that a fracture is present. A plaster of Paris cast is applied to the arm, and the nurse provides instructions to the mother regarding cast care at home. Which of the following instructions would the nurse provide to the mother? 1. "The cast should be dry in about 6 hours." 2. "The cast is water resistant, so the child is able to take a bath or a shower." 3. "The cast will not mold to the body and should heal the fracture in no time at all." 4. "The cast needs to be kept dry because, when wet, it will begin to disintegrate."

"The cast needs to be kept dry because, when wet, it will begin to disintegrate." Rationale: Plaster of Paris is a heavier material than that used in a synthetic cast. It molds easily to the extremity and is less expensive than a synthetic cast. It takes about 24 hours to dry, and drying time could be longer depending on the size of the cast. Plaster of Paris is not water resistant, and when wet, it will begin to disintegrate.

316. A 4-year-old child sustains a fall at home and is brought to the emergency department by the mother. Following x-ray examination, it has been determined that the child has a fractured arm, and a plaster cast is applied. The nurse provides instructions to the mother regarding cast care for the child. Which statement by the mother indicates a need for further instruction? 1. "The cast will feel warm when it is dried." 2. "If the cast becomes wet, a fan may be used to dry the cast." 3. "I need to call the health care provider if any blood or drainage appears on the cast." 4. "I can apply ice to the casted area to prevent swelling."

"The cast will feel warm when it is dried." Rationale: Once the cast dries, the cast will sound hollow and will be cool (not warm) to touch. A fan can be directed toward the cast to facilitate drying. The mother must be instructed to call the health care provider if any blood or drainage appears on the cast. Ice can be applied to the casted area to prevent swelling.

195. A nursing instructor asks a student nurse to describe osteogenic sarcoma. Which statement by the student indicates the need to further research the disease? 1. "The femur is the most common site of this sarcoma." 2. "The child does not experience pain at the primary tumor site." 3. "If a weight-bearing limb is affected, then limping is a clinical manifestation." 4. "The symptoms of the disease during the early stage are almost always attributed to normal growing pains."

"The child does not experience pain at the primary tumor site." Rationale: Osteogenic sarcoma is the most common bone tumor in children. A clinical manifestation of osteogenic sarcoma is progressive, insidious, intermittent pain at the tumor site. By the time these children receive medical attention, they may be in considerable pain from the tumor. Options 1, 3, and 4 are accurate regarding osteogenic sarcoma.

61. A woman who is 6 weeks' pregnant tells the nurse that she is worried that the baby might have spina bifida because of a family history. The nurse's BEST response is: "There is no genetic basis for the defect." "Prenatal detection is not possible yet." "Chromosome studies done on amniotic fluid can diagnose the defect prenatally." "The concentration of alpha-fetoprotein in amniotic fluid can indicate the presence of the defect prenatally."

"The concentration of alpha-fetoprotein in amniotic fluid can indicate the presence of the defect prenatally." The origin of neural tube defects is unknown, but it appears to have a multifactorial inheritance pattern. Prenatal detection is possible through amniotic fluid or chorionic villi sampling. There is no chromosome study at this time. Fetal ultrasound and elevated concentrations of alpha-fetoprotein in amniotic fluid may indicate the presence of anencephaly or myelomeningocele.

307. A nurse is providing information to the mother of a child with nephrotic syndrome regarding the edematous appearance of the child. Which of the following statements should the nurse make to the mother? 1. "Dress the child in loose-fitting clothing to hide the extra weight." 2. "Children always look a little bit fat, so don't be concerned." 3. "The fluid retention should be controlled by medication and diet." 4. "The child will always have this appearance, and preparing the child for the body image change is important."

"The fluid retention should be controlled by medication and diet." Rationale: Most children experience remission with treatment and corticosteroids. Diuretics also may be a component of the treatment plan, and a restricted sodium diet is recommended. It is important to give the parent information in a matter-of-fact manner and address the issue that is the parent's concern. Options 1, 2, and 4 are inaccurate and inappropriate statements to the mother.

237. A nurse has reinforced prior teaching of a school-age child who was given a brace to wear for the treatment of scoliosis. The nurse interprets that the client has not fully understood the information presented if the child makes which statement? 1. "I will wear my brace under my clothes." 2. "I will do back exercises at least five times a week." 3. "I will wear my brace whenever I am not sleeping." 4. "This brace will correct my curve."

"This brace will correct my curve." Rationale: Bracing can halt the progression of most curvatures, although it is not curative for scoliosis. The statements in options 1, 2, and 3 represent correct understanding on the part of the child.

214. A nursing student is asked to discuss juvenile idiopathic arthritis (JIA) at a clinical conference scheduled for the end of the clinical day. Which statement by the nursing student indicates the need to further research this disorder? 1. "The cause of this disease is unknown." 2. "JIA most often occurs by age of 10 years." 3. "This disease is twice as likely to occur in boys rather than girls." 4. "Clinical manifestations include morning stiffness and painful, stiff, swollen joints."

"This disease is twice as likely to occur in boys rather than girls." Rationale: JIA is twice as likely to occur in girls as in boys. The cause of JIA is unknown. JIA has two peak ages of onset: between 1 and 3 years of age and between 8 and 10 years of age. This autoimmune inflammatory disease causes painful inflammation of joints.

238. A nurse is reinforcing instructions to the parents of a child with a hernia regarding measures that will promote reducing the hernia. The nurse determines that the parents understand these measures if they state which of the following? 1. "We will be sure to give our child a Fleet enema every day to prevent constipation." 2. "We will make sure that our child participates in physical activity every day." 3. "We will provide comfort measures to reduce any crying periods by our child." 4. "We will encourage our child to cough every few hours on a daily basis."

"We will provide comfort measures to reduce any crying periods by our child." Rationale: A warm bath and comfort measures to reduce crying periods are all simple measures to promote reducing a hernia. Coughing and crying increase the strain on the hernia. Likewise, physical activities and enemas of any type would increase the strain on the hernia.

230. Which statement should the nurse include when providing safety instructions to the parents of an infant with a diagnosis of hydrocephalus? 1. "When picking up your infant, support the infant's neck and head with the open palm of your hand." 2. "Feed your infant in a side-lying position." 3. "Place a helmet on your infant when in bed." 4. "Hyperextend your infant's head with a rolled blanket under the neck area."

"When picking up your infant, support the infant's neck and head with the open palm of your hand." Rationale: Hydrocephalus is a condition characterized by an enlargement of the cranium caused by an abnormal accumulation of cerebrospinal fluid within the cerebral ventricular system. This characteristic causes the increase in the weight of the infant's head. The infant's head becomes top heavy. Supporting the infant's head and neck when picking it up prevents hyperextension of the neck area and the infant from falling backward. The infant should be fed with the head elevated for proper motility of food processing. A helmet could suffocate an unattended infant during rest and sleep times, and hyperextension of the infant's head can put pressure on the neck vertebrae, causing injury.

5. The nurse is caring for a boy with probable intussusception. He had diarrhea before admission; but, while waiting for administration of air pressure to reduce the intussusception, he passes a normal brown stool. What is the most appropriate nursing action? 1. Notify physician 2. Measure abdominal girth 3. Auscultate for bowel sounds 4. Take vital signs, including blood pressure

1 Notify physician Passage of a normal stool indicates that the intussusception has resolved. Notification of the physician is essential to determine whether a change in treatment plan is indicated. These actions may be indicated, but the physician still should be notified as to the change in status. These actions may be indicated, but the physician still should be notified as to the change in status. These actions may be indicated, but the physician still should be notified as to the change in status.

8. A child is admitted to the hospital and diagnosed with ulcerative colitis. The nurse should evaluate the patient for which associated symptoms? (Select all that apply.) 1. Rectal bleeding 2. Anal lesions 3. Diarrhea 4. Joint pain 5. Anorexia

1, 3, 4, 5. Rectal bleeding Diarrhea Joint pain Anorexia

3. What should the nurse consider when providing support to a family whose infant has just been diagnosed with biliary atresia? 1. Liver transplantation may be needed eventually. 2. Death usually occurs by 6 months of age. 3. Prognosis for a full recovery is excellent. 4. Children with surgical correction live normal lives.

1. Liver transplantation may be needed eventually. Approximately 80% to 90% of children with biliary atresia will require liver transplantation. If untreated, death will usually occur by 2 years of age. Long-term survival is possible with surgical intervention. Liver transplantation is usually required. Even with surgical intervention, most progress to liver failure and require transplantation.

36. What is most descriptive of rhabdomyosarcoma? 1. Most common sites are head and neck 2. Common hereditary neoplasm of childhood 3. Most common bone tumor of childhood 4. Benign tumor; not commonly found in children

1. Most common sites are head and neck Although striated muscle fibers from which this tumor arises can be found anywhere in the body, the most common sites are the head and neck. It is not known to be hereditary. It arises from skeletal muscle tissue, not bone. It is highly malignant.

110. A young boy has just been diagnosed with pseudohypertrophic (Duchenne) muscular dystrophy. His care should include which of the following? Recommend genetic counseling. Explain that the disease is easily treated. Suggest ways to limit use of muscles. Assist family in finding a nursing facility to provide his care.

1. Recommend genetic counseling. Pseudohypertrophic (Duchenne) muscular dystrophy is inherited as an X-linked recessive gene. Genetic counseling is recommended for parents, female siblings, maternal aunts, and their female offspring. 2. No effective treatment exists at this time for childhood muscular dystrophy. 3. Maintaining optimum function of all muscles for as long as possible is the primary goal. It has been found that children who remain as active as possible are able to avoid wheelchair confinement for a longer time. 4. Finding a nursing facility is inappropriate at the time of diagnosis. When the child becomes increasingly incapacitated, the family may consider home-based care, a skilled nursing facility, or respite care to provide the necessary care.

225. A nurse is monitoring a child following a tonsillectomy. Which finding may indicate that the child is bleeding? 1. Restlessness 2. A decreased pulse rate 3. Complaints of discomfort 4. An elevation in blood pressure (BP)

1. Restlessness Rationale: Frequent swallowing, restlessness, a fast and thready pulse, and vomiting bright red blood are signs of bleeding. An elevated BP is not an indication of bleeding. Complaint of discomfort is an expected finding following a tonsillectomy.

157. After a tonsillectomy, which of the following fluid or food items would be appropriate to offer to the child? 1. Yellow Jell-O 2. Cold ginger ale 3. Vanilla pudding 4. Cherry Popsicle

1. Yellow Jell-O Rationale: After a tonsillectomy, clear, cool liquids should be administered. Citrus, carbonated, and extremely hot or cold liquids need to be avoided, because they may irritate the throat. Milk and milk products (pudding) are avoided, because they coat the throat and cause the child to clear the throat, thus increasing the risk of bleeding. Red liquids need to be avoided, because they give the appearance of blood if the child vomits.

18. The nurse is caring for a child with Wilms' tumor. The MOST important nursing intervention before surgery is to: 1. avoid abdominal palpation. 2. closely monitor arterial blood gases. 3. prepare child/family for long-term dialysis. 4. prepare child/family for renal transplantation.

1. avoid abdominal palpation. Wilms' tumors are encapsulated. It is extremely important to avoid any palpation of the mass to minimize the risk of dissemination of cancer cells to adjacent and other sites. This is not indicated before this abdominal surgery. This is not indicated unless both kidneys have to be removed. This option is considered a last resort. If both kidneys are involved, preoperative radiation and/or chemotherapy are used to minimize the size of the tumor. Renal transplantation is a last resort if both kidneys need to be removed and a compatible living donor exists.

17. The nurse should recognize which laboratory value as being abnormal? 1. pH: 4 2. Specific gravity: 1.020 3. Protein level: absent 4. Glucose level: absent

1. pH: 4 The expected pH is 4.8 to 7.8. This is within the normal specific gravity range of 1.016 to 1.022. Protein should not be present in the urine. It would indicate an abnormality in glomerular filtration. Glucose should not be present. If present, it could indicate diabetes mellitus, glomerulonephritis, or a response to infusion of fluids with high glucose concentrations.

11. A 2-month-old breastfed infant is successfully rehydrated with oral rehydration solutions for acute diarrhea. Instructions to the mother about breastfeeding should include to: 1. continue breastfeeding. 2. stop breastfeeding until breast milk is cultured. 3. stop breastfeeding until diarrhea is absent for 24 hours. 4. express breast milk and dilute with sterile water before feeding.

1.Continue breastfeeding Breastfeeding should continue. Culturing the breast milk is not necessary. Breastfeeding can continue along with oral rehydration solution to replace the ongoing fluid loss caused by the diarrhea. Breast milk should not be diluted.

12. Which statement best describes Hirschsprung's disease? 1. The colon has an aganglionic segment. 2. There is passage of excessive amounts of meconium in the neonate. 3. It results in excessive peristaltic movements within the gastrointestinal tract. 4. It results in frequent evacuation of solids, liquid, and gas.

1.The colon has an aganglionic segment Hirschsprung's disease is a mechanical obstruction caused by a lack of motility of a segment of the intestine resulting from the lack of ganglion. It is associated with an inability to pass meconium or feces if the child is older. There is a lack of peristalsis in the affected segment, which interferes with the evacuation of solid waste. There is a lack of peristalsis in the affected segment, which interferes with the evacuation of solid waste.

245. A nurse is preparing to feed a 1-year-old hospitalized child. The nurse prepares the amount of formula to be given to this child, knowing that generally a 1-year-old consumes approximately: 1. 90 mL per feeding 2. 100 mL per feeding 3. 175 mL per feeding 4. 380 mL per feeding

175 mL per feeding Rationale: A 1-year-old child consumes approximately 175 mL (6 ounces) of formula per feeding. Options 1, 2, and 4 are incorrect.

7. An infant with neurologic impairment and delay is receiving several medications. A proton pump inhibitor is one of the medications the infant is receiving. Which medication(s) is/are proton pump inhibitor(s)? (Select all that apply.) 1. Ranitidine (Zantac) 2. Omeprazole (Prilosec) 3. Pantoprazole (Protonix) 4. Glycopyrrolate (Robinul) 5. Bethanechol (Urecholine)

2, 3. Omeprazole (Prilosec) Pantoprazole (Protonix) Omeprazole (Prilosec) uses a proton pump inhibitor that blocks the action of acid- producing cells. Pantoprazole (Protonix) uses a proton pump inhibitor that blocks the action of acid-producing cells. Ranitidine (Zantac) is a histamine-2 (H2) receptor blocker, not a proton pump inhibitor. Ranitidine (Zantac) inhibits the action of histamine at the H2 receptor site in the stomach that results in the inhibition of gastric acid secretion. Glycopyrrolate (Robinul) is an anticholinergic agent that is used to inhibit excessive salivation. Bethanechol (Urecholine) is a prokinetic drug and remains controversial in use.

186. A 4-year-old child sustains a fall at home and is brought to the emergency department by the mother. After an x-ray, it is determined that the child has a fractured arm, and a plaster cast is applied. The nurse provides instructions to the mother regarding cast care for the child. Which statement by the mother indicates the need for further instructions? 1. "The cast may feel warm as it dries." 2. "I can use lotion or powder around the cast edges to relieve itching." 3. "A small amount of white shoe polish can touch up a soiled white cast." 4. "If the cast becomes wet, a blow-dryer set on the cool setting may be used to dry it."

2. "I can use lotion or powder around the cast edges to relieve itching." Rationale: The mother needs to be instructed to not use lotion or powders on the skin around the cast edges or inside the cast, because they can become sticky or caked and cause skin irritation. Options 1, 3, and 4 are appropriate instructions.

180. An 18-month-old child is being discharged after surgical repair of hypospadias. Which postoperative nursing care measure should the nurse stress to the parents as they prepare to take this child home? 1. Leave diapers off to allow the site to heal. 2. Avoid tub baths until the stent has been removed. 3. Encourage toilet training to ensure that the flow of urine is normal. 4. Restrict the fluid intake to reduce urinary output for the first few days.

2. Avoid tub baths until the stent has been removed. Rationale: After hypospadias repair, the parents are instructed to avoid giving the child a tub bath until the stent has been removed to prevent infection. Diapers are placed on the child to prevent the contamination of the surgical site. Toilet training should not be an issue during this stressful period. Fluids should be encouraged to maintain hydration.

179. A nurse collects a urine specimen preoperatively from a child with epispadias who is scheduled for surgical repair. The nurse reviews the child's record for the laboratory results of the urine test and would most likely expect to note which of the following? 1. Hematuria 2. Bacteriuria 3. Glucosuria 4. Proteinuria

2. Bacteriuria Rationale: Epispadias is a congenital defect that involves the abnormal placement of the urethral orifice of the penis. In clients with this condition, the urethral opening is located anywhere on the dorsum of the penis. This anatomical characteristic leads to the easy access of bacterial entry into the urine. Options 1, 3, and 4 are not characteristically noted with this condition.

34. Which statement is true concerning osteogenesis imperfecta? 1. It is easily treated. 2. It is an inherited disorder. 3. Later-onset disease usually runs a more difficult course. 4. Braces and exercises are of no therapeutic value.

2. It is an inherited disorder. It is a lifelong problem caused by defective bone mineralization, abnormal bone architecture, and increased susceptibility to fracture. Osteogenesis imperfecta is an inherited disorder. The type of disease determines the course it will take. Lightweight braces and splints can help support limbs and fractures.

155. The appropriate child position after a tonsillectomy is which of the following? 1. Supine position 2. Side-lying position 3. High Fowler's position 4. Trendelenburg's position

2. Side-lying position Rationale: The child should be placed in a semi-prone or side-lying position after tonsillectomy to facilitate drainage. Options 1, 3, and 4 will not achieve this goal.

16. Management of the child with a peptic ulcer often includes: 1. offering milk at frequent intervals. 2. administering proton pump inhibitors. 3. administering antacids 1 and 3 hours before meals and at bedtime. 4. coping with stress and adjusting to chronic illness.

2. administering proton pump inhibitors Milk is not beneficial in the management of peptic ulcer disease. Proton pump inhibitors block the production of acid. They are well tolerated and have infrequent side effects. Proton pump inhibitors are more effective than antacids. Coping with stress is beneficial, but peptic ulcer disease is treatable.

32. A 3-year-old child is scheduled for surgery to remove a Wilms' tumor from one kidney. The parents ask the nurse about what treatments, if any, will be necessary after recovery from surgery. The nurse's best response is: 1. "No additional treatments are usually necessary." 2. "Chemotherapy may be necessary." 3. "Chemotherapy with or without radiation therapy is indicated." 4. "Kidney transplant is indicated eventually."

3. "Chemotherapy with or without radiation therapy is indicated." Additional therapy is indicated after the tumor is removed. Radiation therapy may be necessary. This determination will be made based on the histologic pattern of the tumor. Chemotherapy with or without radiation therapy is usually indicated. Most children with Wilms' tumor do not require renal transplants.

6. What diet would be appropriate for the child with celiac disease? 1. Salt-free 2. Phenylalanine-free 3. Low gluten 4. High calories, low protein, low fat

3. Low gluten The diet does not have to be salt free. Low phenylalanine is indicated in phenylketonuria. Celiac disease is characterized by intolerance to gluten, the protein found in wheat, barley, rye, and oats. A low-gluten diet is indicated. Diet should be high in calories and protein and low in fat.

20. A 6-year-old child with acute renal failure is being transferred out of the intensive care unit. Considering their diagnoses, which child would be the MOST appropriate roommate for this child? 1. 6-year-old child with pneumonia 2. 4-year-old child with gastroenteritis 3. 5-year-old child who has a fractured femur 4. 7-year-old child who had surgery for a ruptured appendix

3. 5-year-old child who has a fractured femur These children have potentially infectious disease processes. These children have potentially infectious disease processes. The 5-year-old orthopedic patient would be the best choice for a roommate. This child does not have an illness of viral or bacterial origin. These children have potentially infectious disease processes.

224. Following tonsillectomy, which of the health care provider's prescriptions would the nurse question? 1. Monitor vital signs. 2. Monitor for bleeding. 3. Allow ice cream when awake. 4. Offer clear, cool liquids when awake.

3. Allow ice cream when awake. Rationale: Clear, cool liquids are encouraged. Milk and milk products are avoided initially because they coat the throat, which causes the child to clear the throat, increasing the risk of bleeding. Options 1 and 2 are important nursing interventions following any type of surgery.

142. The nurse is planning care for a child with acute glomerulonephritis. The nurse should report which finding to the primary care provider?

blood pressure of 140/92 mm Hg Rationale: The elevated blood pressure may indicate hypertension, which is a serious complication of acute glomerulonephritis.

150. Which of the following is the most appropriate location for assessing the pulse of an infant who is less than 1 year old? 1. Radial 2. Carotid 3. Brachial 4. Popliteal

3. Brachial Rationale: To assess a pulse in an infant (i.e., a child <1 year old), the pulse is checked at the brachial artery. The infant's relatively short, fat neck makes palpation of the carotid artery difficult. The popliteal and radial pulses are also difficult to palpate in an infant.

170. A nurse reinforces instructions to the mother about dietary measures for a 5-year-old child with lactose intolerance. The nurse tells the mother that which of the following supplements will be required as a result of the need to avoid lactose in the diet? 1. Fats 2. Zinc 3. Calcium 4. Thiamine

3. Calcium Rationale: Lactose intolerance is the inability to tolerate lactose, which is the sugar that is found in dairy products. Removing milk from the diet can provide relief from symptoms. Additional dietary changes may be required to provide adequate sources of calcium and, if the child is an infant, protein and calories.

168. A nurse reviews the record of an infant who is seen in the clinic. The nurse notes that a diagnosis of esophageal atresia with tracheoesophageal fistula (TEF) is suspected. The nurse expects to note which most likely clinical manifestation of this condition in the medical record? 1. Incessant crying 2. Coughing at nighttime 3. Choking with feedings 4. Severe projectile vomiting

3. Choking with feedings Rationale: Any child who exhibits the "3 Cs"—coughing and choking during feedings and unexplained cyanosis—should be suspected of having TEF. Options 1, 2, and 4 are not specifically associated with TEF.

10. A school-age child with acute diarrhea and mild dehydration is being given oral rehydration solution (ORS). The child's mother calls the clinic nurse because he is also occasionally vomiting. The nurse should recommend: 1. bringing the child to the hospital for intravenous fluids. 2. alternating giving the child ORS and carbonated drinks. 3. continuing to give the child ORS frequently in small amounts.

3. Continuing to give the child ORS frequently in small amounts A school-age child with mild dehydration can be rehydrated safely at home with oral solutions. Carbonated drinks should not be given to the child. They may have high carbohydrate content and contain caffeine, which is a diuretic. Vomiting is not a contraindication to the use of ORS unless it is severe. The mother should continue to give the ORS in small amounts and at frequent intervals. NPO status is not indicated. Small, frequent intake of ORS is recommended.

164. A mother of a 6-year-old child with type 1 diabetes mellitus calls the clinic nurse and tells the nurse that the child has been sick. The mother reports that she checked the child's urine and it showed positive ketones. Which of the following would the nurse instruct the mother to do? 1. Hold the next dose of insulin. 2. Come to the clinic immediately. 3. Encourage the child to drink liquids. 4. Administer an additional dose of regular insulin.

3. Encourage the child to drink liquids. Rationale: When the child is sick, the mother should test for urinary ketones with each voiding. If ketones are present, liquids are essential to help with clearing them. The child should be encouraged to drink liquids. It is not necessary to bring the child to the clinic immediately, and insulin doses should not be adjusted or changed.

198. A nurse is assigned to care for an infant with cryptorchidism. The nurse anticipates that diagnostic studies will be prescribed to evaluate: 1. DNA synthesis 2. Babinski reflex 3. Kidney function 4. Chromosomal analysis

3. Kidney function Rationale: Cryptorchidism may be the result of hormone deficiency, intrinsic abnormality of a testis, or a structural problem. Diagnostic tests would assess kidney function, because the kidneys and testes arise from the same germ tissue. Babinski's reflex tests neurological function and is unrelated to this diagnosis. DNA synthesis and a chromosomal analysis are also unrelated to this diagnosis.

160. A child with a diagnosis of a hernia has been scheduled for a surgical repair in 2 weeks. The nurse reinforces instructions to the parents about the signs of possible hernial strangulation. The nurse tells the parents that which of the following signs would require health care provider (HCP) notification by the parents? 1. Fever 2. Diarrhea 3. Vomiting 4. Constipation

3. Vomiting Rationale: The parents of a child with a hernia need to be instructed about the signs of strangulation. These signs include vomiting, pain, and an irreducible mass. The parents should be instructed to contact the HCP immediately if strangulation is suspected. Fever, diarrhea, and constipation are not associated with strangulation of a hernia.

22. An advantage of continuous cycling peritoneal dialysis (CCPD) or continuous ambulatory peritoneal dialysis (CAPD) for adolescents that require dialysis is that: 1. hospitalization is only required several nights per week. 2. dietary restrictions are no longer necessary. 3. adolescents can carry out procedures themselves. 4. insertion of catheter does not require surgical placement.

3. adolescents can carry out procedures themselves. Procedure can be done at home. Dietary restrictions are still required but are less strict. This type of dialysis provides the most independence for adolescents with ESRD and their families. Adolescents can carry out the procedure themselves. The catheter is surgically implanted in the abdominal cavity.

15. What should the nurse include when teaching an adolescent with Crohn's disease? 1. Preventing spread of illness to others and nutritional guidance 2. Adjusting to chronic illness and preventing spread of illness to others 3. Coping with stress and adjusting to chronic illness 4. Nutritional guidance and preventing constipation

3.Coping with stress and adjusting to chronic illness Nutritional guidance is necessary, but Crohn's disease is not infectious. Adjustment to chronic illness is necessary, but Crohn's disease is not infectious. Crohn's disease is a chronic disease with life-threatening/life-altering complications. The nursing interventions include helping the child cope with stress and learn how to adjust to the illness. Nutritional guidance is necessary, but constipation is not an issue.

181. The parents of a newborn have been told that their child was born with bladder exstrophy, and the parents ask the nurse about this condition. The nurse bases the response on knowledge that this condition is: 1. A hereditary disorder that occurs in every other generation 2. Caused by the use of medications taken by the mother during pregnancy 3. A condition in which the urinary bladder is abnormally located in the pelvic cavity 4. An extrusion of the urinary bladder to the outside of the body through a defect in the lower abdominal wall

4. An extrusion of the urinary bladder to the outside of the body through a defect in the lower abdominal wall Rationale: Bladder exstrophy is a congenital anomaly that is characterized by the extrusion of the urinary bladder to the outside of the body through a defect in the lower abdominal wall. The cause is unknown, and there is a higher incidence among males. Options 1, 2, and 3 are not characteristics of this disorder.

108. Therapeutic management of a child with tetanus includes the administration of which of the following? Nonsteroidal antiinflammatory drugs (NSAIDs) to reduce inflammation Muscle stimulants to counteract muscle weakness Bronchodilators to prevent respiratory complications Antibiotics to control bacterial proliferation at site of injury

4. Antibiotics to control bacterial proliferation at site of injury Antibiotics are administered to control the proliferation of the vegetative forms of the organism at the site of infection. 1. Tetanus toxin acts at the myoneural junction to produce muscular stiffness and lowers the threshold for reflex excitability. NSAIDs are not routinely used. 2. Sedatives or muscle relaxants are used to help reduce titanic spasm and prevent seizures. 3. Respiratory status is carefully evaluated for any signs of distress, since muscle relaxants, opioids, and sedatives that may be prescribed may cause respiratory depression. Bronchodilators would not be used unless specifically indicated.

159. A nurse is caring for a child with a diagnosis of intussusception. Which of the following symptoms would the nurse expect to note in this child? 1. Watery diarrhea 2. Ribbon-like stools 3. Profuse projectile vomiting 4. Blood and mucus in the stools

4. Blood and mucus in the stools Rationale: The child with intussusception classically presents with severe abdominal pain that is crampy and intermittent and that causes the child to draw in his or her knees to the chest. Vomiting may be present, but it is not projectile. Bright red blood and mucus are passed through the rectum and commonly described as currant jelly-like stools. Ribbon-like stools are not a manifestation of this disorder.

33. Which is a secondary effect when a child experiences decreased muscle strength, tone, and endurance from immobilization? 1. Increased metabolism 2. Increased venous return 3. Increased cardiac output 4. Decreased exercise tolerance

4. Decreased exercise tolerance Metabolism decreases during periods of immobility. There is decreased venous return because of decreased muscle activity. There is decreased cardiac output. Muscle disuse leads to tissue breakdown and loss of muscle mass. It may take weeks or months to recover.

4. The nurse assesses the neonate immediately after birth. A tracheoesophageal fistula should be suspected if what condition is present? 1. Jaundice 2. Clubfeet 3. Absence of sucking 4. Excessive amount of frothy saliva in the mouth

4. Excessive amount of frothy saliva in the mouth Jaundice is not usually associated with a tracheoesophageal fistula. Clubfeet are not usually associated with a tracheoesophageal fistula. The infant is able to suck, just not manage the secretions. Excessive salivation and drooling indicates tracheoesophageal fistulas. With a fistula the child has difficulty managing the secretions, causing choking, coughing, and cyanosis.

153. A nurse is assisting with data collection from an infant who has been diagnosed with hydrocephalus. If the infant's level of consciousness diminishes, a priority intervention is: 1. Taking the apical pulse 2. Taking the blood pressure 3. Testing the urine for protein 4. Palpating the anterior fontanel

4. Palpating the anterior fontanel Rationale: A full or bulging anterior fontanel indicates an increase in cerebrospinal fluid collection in the cerebral ventricle. Apical pulse and blood pressure changes and proteinuria are not specifically associated with increasing cerebrospinal fluid in the brain tissue in an infant.

97. Pica refers to which of the following? Overeating of nonnutritive food substances Refusal to eat proper amounts of food despite its availability Obsessive ingestion of unusual foods that persists over time Compulsive and excessive ingestion of both food and nonfood substances

4. Pica is the compulsive and excessive ingestion of both food and nonfood substances. Food picas include the excessive eating of ordinary foods or unprepared food substances, such as coffee grounds or uncooked cereals. Nonfood picas include the ingestion of substances such as clay, soil, stones, laundry starch, paint chips, ice, hair, paper, rubber, and feces. 1. Pica also includes the eating of nonfood substances. 2. Eating, not refusing to ingest, is the issue in pica. 3. Nonfood substances are also involved.

13. A child has a nasogastric (NG) tube after surgery for acute appendicitis. The purpose of the NG tube is to: 1. maintain electrolyte balance. 2. maintain an accurate record of output. 3. prevent the spread of infection. 4. prevent abdominal distention.

4. Prevent abdominal distention The NG tube may adversely affect electrolyte balance by removing stomach secretions. NG drainage is one part of the child's output. The nurse would need to incorporate the NG drainage with other output. There is no relationship to the spread of infection. The NG tube is used to maintain gastric decompression until the return of intestinal activity.

A nurse is assisting a health care provider (HCP) during the examination of an infant with hip dysplasia. The HCP performs the Ortolani maneuver. Which of the following best describes the action/purpose of the Ortolani maneuver? 1. Determining the extent of range of motion 2. Checking for asymmetry on the affected side 3. Pushing the unstable femoral head out of the acetabulum 4. Reducing the dislocated femoral head back into the acetabulum.

4. Reducing the dislocated femoral head back into the acetabulum. Rationale: With the Ortolani maneuver, the examiner reduces the dislocated femoral head back into the acetabulum. A positive Ortolani maneuver is a palpable clunk as the femoral head moves over the acetabular ring. Options 1 and 2 are data collection techniques for the identification of the clinical manifestations of hip dysplasia, but they do not describe the Ortolani maneuver. When performing the Barlow maneuver, the examiner pushes the unstable femoral head out of the acetabulum.

9. When evaluating the extent of an infant's dehydration, the nurse should recognize that the symptoms of severe dehydration include: 1. tachycardia, decreased tears, 5% weight loss. 2. normal pulse and blood pressure, intense thirst. 3. irritability, moderate thirst, normal eyes and fontanel. 4. tachycardia, parched mucous membranes, sunken eyes and fontanel.

4. Tachycardia, parched mucous membranes, sunken eyes and fontanel In severe dehydration there is a 15% weight loss in infants. Tachycardia, orthostatic hypotension and shock, and intense thirst would be expected. The infant would be extremely irritable, with sunken eyes and fontanel. These are the symptoms of severe dehydration.

151. A child has a basilar skull fracture. Which of the following health care provider's prescriptions should the nurse question? 1. Restrict fluid intake. 2. Insert an indwelling urinary catheter. 3. Keep an intravenous (IV) line patent. 4. Suction via the nasotracheal route as needed.

4. Suction via the nasotracheal route as needed. Rationale: Nasotracheal suctioning is contraindicated in a child with a basilar skull fracture. Because of the location of the injury, the suction catheter may be introduced into the brain. Fluids are restricted to prevent fluid overload. The child may require a urinary catheter for the accurate monitoring of intake and output. An IV line is maintained to administer fluids or medications, if necessary.

14. Therapeutic management of the child with an inflammatory bowel disease (IBD) includes a diet that has which component? 1. Low protein 2. Low calorie 3. High fiber 4. Vitamin supplements

4. Vitamin supplements A high-protein, high-calorie diet is needed to help correct nutrition deficits. A high-protein, high-calorie diet is needed to help correct nutrition deficits. A high-fiber diet is not recommended for IBD. Even small amounts of bran have been associated with a worsening of the child's condition. Multivitamins, iron, and folic acid supplementation are recommended.

19. A toddler is hospitalized with acute renal failure secondary to severe dehydration. The nurse should assess the child for what possible complication? 1. Hypotension 2. Hypokalemia 3. Hypernatremia 4. Water intoxication

4. Water intoxication The child needs to be monitored for hypertension. Hyperkalemia is a concern in acute renal failure. Hyponatremia may develop in acute renal failure. The child with acute renal failure has the tendency to develop water intoxication with hyponatremia. Control of water balance requires careful monitoring of intake, output, body weight, and electrolytes.

109. An adolescent has sustained a spinal cord injury. The first stage, known as spinal shock syndrome, is characterized by: increasing spasticity. spinal reflex activity. symptoms of hypertension. flaccid paralysis below level of damage.

4. flaccid paralysis below level of damage. Reflexes are absent at or below the cord lesion. 1. There is flaccidity or limpness of the involved muscles. 2. Spinal reflex activity occurs in the second stage. 3. Symptoms of hypotension occur.

35. An appropriate nursing intervention when caring for the child with chronic osteomyelitis is to: 1. provide active range-of-motion exercises for the affected extremity. 2. administer antibiotics with meals. 3. encourage frequent ambulation. 4. move and turn the child carefully and gently to minimize pain.

4. move and turn the child carefully and gently to minimize pain. Active range of motion is contraindicated until pain has subsided. Pain medication should be administered as needed. Ambulation is contraindicated until pain has subsided. Osteomyelitis is extremely painful. Movement is carried out only as needed and then carefully and gently.

21. An important nursing consideration when caring for a child with end-stage renal disease (ESRD) is that: 1. children with ESRD usually adapt well to the minor inconveniences of treatment. 2. children with ESRD require extensive support until they outgrow the condition. 3. multiple stresses are placed on children with ESRD and their families until the illness is cured. 4. multiple stresses are placed on children with ESRD and their families because the children's lives are maintained by drugs and artificial means.

4. multiple stresses are placed on children with ESRD and their families because the children's lives are maintained by drugs and artificial means. ESRD is a complex disease process that requires substantial medical intervention. ESRD cannot be outgrown. Dialysis is necessary until renal transplantation is performed. ESRD cannot be cured. Dialysis is necessary until renal transplantation is performed. This is a chronic, progressive disease with dependence on technology. Families need to arrange for continuing examinations and procedures that are painful and may require hospitalization.

106. The parents of a child with cerebral palsy ask the nurse if any drugs can decrease their child's spasticity. The nurse's response should be based on which of the following? Anticonvulsant medications are sometimes useful for controlling spasticity. Medications that would be useful in reducing spasticity are too toxic for use with children. Many different medications can be highly effective in controlling spasticity. Implantation of a pump to deliver medication into the intrathecal space decreases spasticity.

4.Implantation of a pump to deliver medication into the intrathecal space decreases spasticity. Baclofen, given intrathecally, is best suited for children with severe spasticity that interferes with activities of daily living and ambulation. 1. Anticonvulsant medications are used when seizures occur in children with cerebral palsy. 2. The intrathecal route decreases the side effects of the drugs that reduce spasticity. 3. Few medications are currently available for the control of spasticity.

290. A nurse in a newborn nursery is told that a newborn with spina bifida (myelomeningocele type) will be transported from the delivery room. The nurse is asked to prepare for the arrival of the newborn. The nurse places which of the following priority items at the newborn's bedside? 1. A blood pressure cuff 2. A rectal thermometer 3. A specific gravity urinometer 4. A bottle of sterile normal saline

A bottle of sterile normal saline Rationale: The newborn with spina bifida is at risk for infection before the closure of the gibbus. A sterile normal-saline dressing is placed over the gibbus to maintain moisture of the gibbus and its contents. This prevents tearing or breakdown of the skin integrity at the site. Blood pressure is difficult to assess during the newborn period and is not the best indicator of infection. Urine concentration is not well developed in the newborn stage of development. A thermometer will be needed to assess the temperature, but in this newborn the priority is to maintain sterile normal-saline dressings over the gibbus.

205. A nurse is caring for a child recently diagnosed with cerebral palsy. The parents of the child ask the nurse about the disorder. The nurse bases the response to the parents on the understanding that cerebral palsy is: 1. An infectious disease of the central nervous system 2. An inflammation of the brain as a result of a viral illness 3. A congenital condition that results in moderate to severe retardation 4. A chronic disability characterized by a difficulty in controlling the muscles

A chronic disability characterized by a difficulty in controlling the muscles Rationale: Cerebral palsy is a chronic disability characterized by difficulty in controlling the muscles as a result of an abnormality in the extrapyramidal or pyramidal motor system. Meningitis is an infectious process of the central nervous system. Encephalitis is an inflammation of the brain that occurs as a result of viral illness or central nervous system infection. Down syndrome is an example of a congenital condition that results in moderate to severe retardation.

266. A 1-year-old child is diagnosed with intussusception. The mother of the child asks the nurse to describe the disorder. The nurse tells the mother that this disorder is: 1. An acute bowel obstruction 2. A condition in which a proximal segment of the bowel prolapses into a distal segment of the bowel 3. A condition in which a distal segment of the bowel prolapses into a proximal segment of the bowel 4. A condition that causes an acute inflammatory process in the bowel

A condition in which a proximal segment of the bowel prolapses into a distal segment of the bowel Rationale: Intussusception occurs when a proximal segment of the bowel prolapses into a distal segment of the bowel. It is a common cause of acute bowel obstruction in infants and young children. It is not an inflammatory process.

295. A licensed practical nurse (LPN) is assigned to assist in caring for a hospitalized child who is receiving a continuous infusion of intravenous (IV) potassium for the treatment of dehydration. The LPN monitors the child closely and notifies the registered nurse if which of the following is noted? 1. Temperature of 100.8° F rectally 2. Weight increase of 0.5 kg 3. A decrease in urine output to 0.5 mL/kg/hr 4. Blood pressure (BP) unchanged from baseline

A decrease in urine output to 0.5 mL/kg/hr Rationale: The priority assessment is to monitor the status of urine output. Potassium should never be administered in the presence of oliguria or anuria. If urine output is less than 1 to 2 mL/kg/hr, potassium should not be administered. A slight elevation in temperature would be expected in a child with dehydration. A weight increase of 0.5 kg is relatively insignificant. A BP that is unchanged is a positive indicator unless the baseline was abnormal. However, there is no information in the question to support such data.

199. A nurse is assigned to care for a child with a compound (open) fracture of the arm that occurred as a result of a fall. The nurse plans care, knowing that this type of fracture involves: 1. The entire bone fractured straight across 2. A greater risk of infection than a simple fracture 3. One side of the bone being broken and the other side being bent 4. The bone being fractured but not producing a break in the skin

A greater risk of infection than a simple fracture Rationale: In a compound (open) fracture, a wound in the skin leads to the broken bone, and there is an added danger of infection. Option 1 describes a transverse fracture. Option 3 describes a greenstick fracture. Option 4 describes a closed or simple fracture.

252. A nurse is assigned to care for a child with a diagnosis of Wilms' tumor. In planning care for the child, the nurse understands that this tumor is: 1. An abdominal tumor 2. A renal tumor 3. A brain tumor 4. A bone tumor

A renal tumor Rationale: Wilms' tumor, or nephroblastoma, is the most common renal tumor in children. Arising from the renal parenchyma of the kidney, this tumor grows very rapidly. It may be present unilaterally and localized, or bilaterally, and sometimes with metastasis to other organs.

165. A nurse is caring for an 18-month-old child who has been vomiting. The appropriate position in which to place the child during naps and sleep time is: 1. A supine position 2. A side-lying position 3. Prone, with the head elevated 4. Prone, with the face turned to the side

A side-lying position Rationale: The vomiting child should be placed in an upright or side-lying position to prevent aspiration. Options 1, 3, and 4 will place the child at risk for aspiration if vomiting occurs.

53. Which statement is most accurate in describing tetanus? Acute infectious disease caused by an exotoxin produced by an anaerobic gram-positive bacillus Inflammatory disease that causes extreme, localized muscle spasm Acute infection that causes meningeal inflammation resulting in symptoms of generalized muscle spasm Disease affecting the salivary gland with resultant stiffness of the jaw

Acute infectious disease caused by an exotoxin produced by an anaerobic gram-positive bacillus Tetanus is an acute, preventable disease caused by an exotoxin produced by an anaerobic spore-forming, gram-positive bacillus, Clostridium tetani. These symptoms are caused by the effect of the toxins becoming fixed on nerve cells. These symptoms are caused by the effect of the toxins becoming fixed on nerve cells. These symptoms are caused by the effect of the toxins becoming fixed on nerve cells

30. What is an appropriate nursing intervention while the child with nephrotic syndrome is confined to bed? 1. Restraining child as necessary 2. Discouraging parents from holding child 3. Doing passive range-of-motion exercises once a day 4. Adjusting activities to child's tolerance level

Adjusting activities to child's tolerance level Restraints should not be used. Parents should be encouraged to hold child. The child should be encouraged to move all extremities while in bed. The child will have a variable level of tolerance for activity. This will also be affected by the labile moods associated with steroid administration. The nurse should assist the family in adjusting activities for the child.

222. A nurse is caring for a child who is scheduled for an appendectomy. When the nurse reviews the health care provider's preoperative prescriptions, which of the following would be questioned? 1. Administer a Fleet enema. 2. Maintain nothing per mouth (NPO) status. 3. Maintain intravenous (IV) fluids as prescribed. 4. Administer preoperative medication on call to the operating room.

Administer a Fleet enema. Rationale: In the preoperative period, enemas or laxatives should not be administered. No heat should be applied to the abdomen because this may increase the chance of perforation secondary to vasodilation. IV fluids would be started and the child would be NPO. Prescribed preoperative medications most likely would be administered on call to the operating room.

96. Which of the following is an advantage of continuous cycling peritoneal dialysis or continuous ambulatory peritoneal dialysis for adolescents who require dialysis? Dietary restrictions are no longer necessary. Hospitalization is only required several nights per week. Adolescents can carry out procedures themselves. Insertion of a catheter does not require surgical placement.

Adolescents can carry out procedures themselves. 3. Continuous cycling peritoneal dialysis or continuous ambulatory peritoneal dialysis provides the most independence for adolescents with end-stage renal disease and their families. Adolescents can carry out the procedure themselves. 1. Dietary restrictions are still required, but are less strict. 2. The procedure can be done at home. 4. The catheter is surgically implanted in the abdominal cavity.

92. Which of the following statements is correct regarding sports injuries during adolescence? Rapidly growing bones, muscles, joints, and tendons offer some protection from unusual strain. The increase in strength and vigor during adolescence helps prevent injuries related to fatigue. More injuries occur during organized athletic competition than during recreational sports participation. Adolescents may not possess the insight and judgment to recognize when an activity is beyond their capabilities.

Adolescents may not possess the insight and judgment to recognize when an activity is beyond their capabilities. Children and adolescents may not possess the insight and judgment to recognize when an activity is beyond their capabilities. Rapidly growing bones, muscles, joints, and tendons are especially vulnerable to unusual strain. The increase in strength and vigor in adolescence may tempt adolescents to overextend themselves. More injuries occur during recreational sports participation than in organized athletic competition.

292. A nurse employed in an emergency department is instructed to monitor a child diagnosed with epiglottitis. The nurse notes that the child is leaning forward with the chin thrust out. The nurse interprets this finding as indicating: 1. The presence of dehydration 2. The presence of pain 3. Extreme fatigue 4. An airway obstruction

An airway obstruction Rationale: Clinical manifestations suggestive of airway obstruction include tripod positioning (leaning forward supported by arms, chin thrust out, mouth open), nasal flaring, tachycardia, a high fever, and sore throat. The data in the question do not relate to options 1, 2, or 3.

315. A nurse is caring for a child with a fracture who is placed in skeletal traction. The nurse monitors for the most serious complication associated with this type of traction by checking for: 1. An increase in the blood pressure 2. A decrease in the urinary output 3. A lack of appetite 4. An elevated temperature

An elevated temperature Rationale: The most serious complication associated with skeletal traction is osteomyelitis, an infection involving the bone. Organisms gain access to the bone systemically or through the opening created by the metal pins or wires used with the traction. Osteomyelitis may occur with any open fracture. Clinical manifestations include complaints of localized pain, swelling, warmth, tenderness, an unusual odor from the fracture site, and an elevated temperature.

257. A nurse provides instructions to the parents of an infant with gastroesophageal reflux (GER) regarding proper positioning to manage reflux. The nurse tells the parents that the infant should be maintained in: 1. A 30-degree angle when supine 2. A 60-degree angle when prone 3. An upright angle 24 hours a day 4. A 20-degree angle when side-lying

An upright angle 24 hours a day Rationale: Proper positioning is an important component of reflux management. Ideally the goal is to maintain the infant in an upright angle 24 hours a day, at a 60-degree angle when supine, and at a 30-degree angle when prone. This position is maintained until the infant remains asymptomatic for 6 weeks

67. Which of the following best describes the cause of most cases of acute glomerulonephritis? Renal vascular anomalies Antecedent streptococcal infection Results from a urinary tract infection Structural anomalies of genitourinary tract

Antecedent streptococcal infection Most cases are postinfectious and have been associated with pneumococcal, streptococcal, or viral infections. Renal vascular anomalies are not associated with acute glomerulonephritis. Urinary tract infections and structural anomalies can result in progressive renal injury, not acute glomerulonephritis.

98. Constipation has recently become a problem for a school-age girl. She is healthy except for seasonal allergies, which are now being successfully treated with antihistamines. In this situation, the constipation is most likely caused by which of the following? Diet Puberty Allergies Antihistamines

Antihistamines Constipation may be associated with drugs such as antihistamines, antacids, diuretics, opioids, antiepileptics, and iron. Because this is the only known change in her habits, the addition of antihistamines is the most likely cause of the constipation. 1. With a change in bowel habits, the presence and role of any recently prescribed medications should be assessed. 2. With a change in bowel habits, the presence and role of any recently prescribed medications should be assessed. 3. With a change in bowel habits, the presence and role of any recently prescribed medications should be assessed.

286. A lethargic, pale child is brought to the health care provider's office with symptoms of periorbital edema and reduced quantity of urine output. The urine is cloudy and smoky in color. The nurse asks the mother if the child has had any recent infections, to which the mother responds that the child had a very sore throat a few weeks ago. The health care provider suspects that the child might have acute poststreptococcal glomerulonephritis. Which of the following laboratory tests would rule out a past streptococcal infection in the child? 1. Urinalysis 2. Throat culture 3. Antistreptolysin titer 4. Creatinine clearance

Antistreptolysin titer Rationale: Option 3 is the only laboratory test that will determine if a streptococcal infection was present. The other options do not relate to a past streptococcal infection. Option 1 will determine if protein is present in the urine, which is present in glomerulonephritis. Option 2 will determine if a current throat infection is present. Option 4 will determine glomerular filtration rate.

273. A nurse is observing a student preparing to suction a pediatric client through a tracheostomy. The nurse intervenes if the student verbalizes to: 1. Limit insertion and suctioning time to 5 seconds. 2. Reoxygenate the child between suction catheter passes. 3. Apply continuous suction when inserting the catheter. 4. Use a twisting motion on the catheter when withdrawing the catheter.

Apply continuous suction when inserting the catheter. Rationale: The nurse would not use continuous suction on the catheter during insertion; suction is applied only when withdrawing the catheter. Options 1, 2, and 4 represent correct interventions regarding this procedure.

312. An adolescent is seen in the emergency department following an athletic injury, and it is suspected that the child sprained an ankle. X-rays are taken, and a fracture has been ruled out. The nurse provides instructions to the adolescent regarding home care for treatment of the sprain and tells the adolescent which of the following? 1. Apply ice to the injured area for a period of 30 minutes every 4 to 6 hours for the first 24 hours. 2. Apply heat to the injured area every 4 hours for the first 48 hours, and then begin to apply ice. 3. Immobilize the extremity and maintain the extremity in a dependent position. 4. Elevate the extremity and maintain strict bedrest for a period of 7 days.

Apply ice to the injured area for a period of 30 minutes every 4 to 6 hours for the first 24 hours. Rationale: To treat a sprain, the injured area should be wrapped immediately to support the joint and control the swelling. Ice is applied to reduce the swelling and should be applied for no longer than 30 minutes every 4 to 6 hours for the first 24 to 48 hours. The joint should be immobilized and elevated, but strict bedrest for a period of 7 days is not required. A dependent position will cause swelling in the affected area.

289. A 4-year-old child is hospitalized for severe gastroenteritis. The child is crying and clinging to the mother. The mother becomes very upset and is afraid to leave the child. Which of the following nursing interventions would be most appropriate to alleviate the child's fears and the mother's anxiety? 1. Reassure the mother that the child will be fine after she leaves. 2. Give the mother the telephone number of the pediatric unit, and tell the mother to call at any time. 3. Ask the mother if she would like to stay overnight with the child. 4. Tell the mother to bring the child's favorite toys the next time she comes to the hospital to visit.

Ask the mother if she would like to stay overnight with the child. Rationale: Although a 4-year-old may already be spending some time away from his or her parents at a day care center or preschool, illness adds a stressor that makes separation more difficult. The only option that addresses the mother's anxiety, while at the same time alleviating the fears of the child is option 3. Options 1, 2, and 4 do not address the fears and anxieties of the mother and child.

81. Which critical element should be included in a plan of care for a pediatric patient who has to be immobilized as part of the course of therapeutic intervention? Make sure that antiemetic therapy has been ordered. Including Braden scale measurement every 4 hours. Asking the patient to help organize individual care into a schedule. Restrict visitors to help maintain patient's environment.

Asking the patient to help organize individual care into a schedule. One of the critical elements for a pediatric patient is to allow them to participate in scheduling their care. This allows for the patient to have "self-care" choices which provides for more control over their environment. Antiemetic therapy and restriction of visitors may not be required and are related to the patient's immediate medical diagnosis. The Braden scale should be implemented but it is typically done once a shift and not every 4 hours.

283. A newborn is transferred to the neonatal intensive care unit with an admitting diagnosis of esophageal atresia accompanied by a distal tracheoesophageal fistula (TEF). When assisting to care for the newborn, the priority concern would be: 1. Pain 2. Infection 3. Aspiration 4. The parents' concerns

Aspiration Rationale: Because TEF manifests itself with regurgitation and coughing, the concern that has the highest priority is aspiration. Although the other problems are an important part of care, the one with the highest concern relates to airway.

39. A nurse is caring for an infant with developmental dysplasia of the hip (DDH). Based on the nurse's knowledge of DDH, which clinical manifestation should the nurse expect to observe? (Select all that apply.) Lordosis Negative Babinski sign Asymmetric thigh and gluteal folds Positive Ortolani and Barlow tests Shortening of limb on affected side

Asymmetric thigh and gluteal folds Positive Ortolani and Barlow tests Shortening of limb on affected side Asymmetric thigh and gluteal folds are a clinical manifestation of DDH and seen from birth to 2 months old. Positive Ortolani and Barlow tests are clinical manifestations of DDH. Ortolani test is the abducting of the thighs to test for hip subluxation or dislocation. Barlow test is the adducting to feel if the femoral head slips out of the socket posterolaterally. Shortening of limb on affected side is another clinical manifestation of DDH. Lordosis is the inward curve of the lumbar spine just above the buttocks and is not a clinical manifestation of DDH. A negative Babinski sign is not a clinical manifestation of DDH. It is a neurologic reflex.

208. An infant is seen in a clinic and is diagnosed with unilateral hip dysplasia. Which finding is associated with this condition? 1. Limited range of motion in the unaffected hip 2. An apparent short femur on the unaffected side 3. Adduction of the affected hip when placed supine with the knees and hips flexed 4. Asymmetry of the gluteal skin folds when the infant is placed prone and the legs are extended against the examining table

Asymmetry of the gluteal skin folds when the infant is placed prone and the legs are extended against the examining table Rationale: Asymmetry of the gluteal folds when the infant is placed prone would be a finding in hip dysplasia in infants beyond the newborn period. Options 1, 2, and 3 are inaccurate assessment findings in this disorder.

309. A nurse is reviewing the health care record of an infant suspected of having unilateral hip dysplasia. Which assessment finding would the nurse expect to note documented in the infant's record regarding this condition? 1. Asymmetric adduction of the affected hip when placed supine with the knees and hips flexed 2. Asymmetry of the gluteal skin folds when the infant is placed prone and the legs are extended against the examining table 3. An apparent short femur on the unaffected side 4. Full range of motion in the affected hip

Asymmetry of the gluteal skin folds when the infant is placed prone and the legs are extended against the examining table Rationale: Asymmetry of the gluteal skin folds when the infant is placed prone and the legs are extended against the examining table is noted in hip dysplasia. Asymmetrical abduction of the affected hip, when an infant is placed supine with the knees and hips flexed, also would be an assessment finding in hip dysplasia in infants beyond the newborn period. An apparent short femur on the affected side is noted, as well as limited range of motion.

216. A nurse is caring for an infant. A urinalysis has been prescribed, and the nurse plans to collect the specimen. The nurse implements which appropriate method to collect the specimen? 1. Catheterizes the infant, using a No. 5 French Foley 2. Attaches a urinary collection device to the infant's perineum 3. Obtains the specimen from the diaper, using a syringe, after the infant voids 4. Monitors the urinary patterns and prepares to collect the specimen into a cup when the infant voids

Attaches a urinary collection device to the infant's perineum Rationale: Although many methods have been used to collect urine from an infant, the most reliable method is the urine collection device. This device is a plastic bag that has an opening lined with adhesive so that it may be attached to the perineum. Urine for certain tests, such as specific gravity, may be obtained from a diaper. Urinary catheterization is not to be done unless specifically prescribed because of the risk of infection. It is not reasonable to monitor urinary patterns and attempt to collect the specimen in a cup when the infant voids.

300. A nursing student caring for a 6-month-old infant is asked to collect a urine specimen from the infant. The student collects the specimen by: 1. Attaching a urinary collection device to the infant's perineum for collection 2. Obtaining the specimen from the diaper by squeezing the diaper after the infant voids 3. Catheterizing the infant using the smallest available French Foley catheter 4. Noting the time of the next expected voiding and preparing to collect the specimen into a cup when the infant voids

Attaching a urinary collection device to the infant's perineum for collection Rationale: Although many methods have been used to collect urine from an infant, the most reliable method is the urine collection device. This device is a plastic bag that has an opening that is lined with adhesive so that it may be attached to the perineum. Urine for certain tests, such as specific gravity, may be obtained from a diaper by collection of the urine with a syringe. Urinary catheterization is not to be done unless specifically prescribed because of the risk of infection. It is not reasonable to try to identify the time of the next voiding to attempt to collect the specimen.

75. A stool specimen from a child with diarrhea shows the presence of neutrophils and red blood cells. This is most suggestive of which of the following conditions? Parasitic infection Fat malabsorption Protein intolerance Bacterial gastroenteritis

Bacterial gastroenteritis Bacterial gastroenteritis is associated with neutrophils or red blood cells in the stool. The stool will be analyzed for ova and parasites when the bacterial and viral cultures are negative. Malabsorption is characterized by a pH of less than 6 and presence of reducing substances. Diarrhea that develops after introduction of cow's milk, fruits, or cereals may be related to protein intolerance.

291. A nurse employed in the emergency department is collecting data on a 7-year-old child with a fractured arm. The child is hesitant to answer questions that the nurse is asking and consistently looks at the parents in a fearful manner. The nurse suspects physical abuse and continues with the data collection procedures. Which of the following findings would most likely assist in verifying the suspicion? 1. Poor hygiene 2. Bald spots on the scalp 3. Lacerations in the anal area 4. Swelling of the genitals

Bald spots on the scalp Rationale: Bald spots on the scalp are most likely to be associated with physical abuse. The most likely findings in sexual abuse include difficulty walking or sitting; torn, stained, or bloody underclothing; pain; swelling or itching of the genitals; and bruises, bleeding, or lacerations in the genital or anal area. Poor hygiene may be indicative of physical neglect.

118. A child in renal failure has hyperkalemia. Which foods should be avoided? Cold cuts, chips, and canned foods Hamburger on a bun and lime Jell-O Spaghetti with meat sauce and breadsticks Bananas, carrots, and green leafy vegetables

Bananas, carrots, and green leafy vegetables Bananas, carrots, and green leafy vegetables are high in potassium. Cold cuts, chips, and canned foods are high in sodium but not necessarily in potassium. A hamburger on a bun and lime Jell-O is an acceptable choice for a low-potassium diet. Spaghetti with meat sauce and breadsticks is an acceptable choice for a low-potassium diet.

275. A nurse is reviewing the record of a child scheduled for a health care provider's visit. Before data collection, the nurse notes documentation that the child has enuresis. Based on this diagnosis, the nurse plans to focus on which of the following when collecting data? 1. Bowel function 2. Bladder function 3. Motor development 4. Nutritional status and weight gain

Bladder function Rationale: Enuresis refers to a condition in which the child is unable to control bladder function, although he or she has reached an age at which control of voiding is expected. Nocturnal enuresis, or bed-wetting, is common in children.

95. The nurse is preparing an adolescent girl for surgery to treat scoliosis. Which of the following should the nurse include? Blood administration may be an option. Ambulation will not be allowed for up to 3 months. Surgery eliminates the need for casting and bracing. Discomfort can be controlled with nonpharmacologic methods.

Blood administration may be an option. Spinal surgery usually involves considerable blood loss. Several options are considered for blood replacement. Ambulation is allowed as soon as possible. Depending on the instrumentation used, most patients are walking by the second or third postoperative day. Casting and bracing are required postoperatively. The child usually has considerable pain for the first few days after surgery. Intravenous opioids should be administered on a regular basis.

135. A nurse is developing a teaching plan for a child with acute poststreptococcal glomerulonephritis. What is the most important point to address in this plan?

Blood pressure monitoring Rationale: can lead to HTN

90. A child, age 10 years, sustained a fracture in the epiphyseal plate of her right fibula when she fell out of a tree. When discussing this injury with her parents, the nurse should consider which of the following? This type of fracture is inconsistent with a fall. Bone growth can be affected by this type of fracture. This is an unusual fracture site in young children. Healing is usually delayed in this type of fracture.

Bone growth can be affected by this type of fracture. Detection of epiphyseal injuries is sometimes difficult, but fractures involving the epiphysis or epiphyseal plate present special problems in determining whether bone growth will be affected. The epiphysis is the weakest point of the long bones. This is a frequent site of damage during trauma. Healing of epiphyseal injuries is usually prompt.

78. The nurse is explaining to a parent how to care for a child with vomiting associated with a viral illness. Which of the following should the nurse include? Give nothing by mouth for 24 hours. Avoid carbohydrate-containing liquids. Brush teeth or rinse mouth after vomiting. Give plain water until vomiting ceases for at least 24 hours.

Brush teeth or rinse mouth after vomiting. It is important to emphasize the need for the child to brush the teeth or rinse the mouth after vomiting to dilute the hydrochloric acid that comes in contact with the teeth. Ad libitum administration of glucose-electrolyte solution to an alert child will help restore water and electrolytes satisfactorily. It is important to include carbohydrates to spare body protein and avoid ketosis.

247. A mother of a 6-year-old-child calls a nurse who lives in the neighborhood and tells the nurse that her child accidentally rubbed waterproof sunscreen in his eyes. The nurse should tell the mother to immediately: 1. Flush the child's eyes for 15 minutes with water. 2. Have the child wipe the eyes with a wet towel. 3. Tell the child to blink continuously to get the sunscreen out of the eye. 4. Call the poison control center.

Call the poison control center. Rationale: Waterproof sunscreen should never be placed near the eyes. Waterproof sunscreen causes severe pain and a chemical burn that can damage the child's vision. Flushing the eyes with water does not stop the burning. The mother should be instructed to call the poison control center and to take the child to the emergency department. Special chemicals will be needed to flush the sunscreen out of the eyes and preserve vision. Wiping the eyes will increase the pain and burning. Blinking will not alleviate the pain or remove the sunscreen from the eyes.

249. A child is to be admitted to the orthopedic unit following a Harrington rod insertion for the treatment of scoliosis. The nurse is assisting in preparing a plan of care for the child. The nurse plans to monitor which priority item in the immediate postoperative period? 1. Pain level 2. Capillary refill, sensation, and motion in all extremities 3. Ability to turn using the logroll technique 4. Ability to flex and extend the lower extremities

Capillary refill, sensation, and motion in all extremities Rationale: When the spinal column is manipulated during surgery, altered neurovascular status is a possible complication; therefore neurovascular assessments including circulation, sensation, and motion should be done every 2 hours. Level of pain and ability to flex and extend the lower extremities are important postoperative assessments but not the priorities of the options provided. Logrolling would be performed by nurses.

105. A 6-month-old infant does not smile, has poor head control, has a persistent Moro reflex, and often gags and chokes while eating. These findings are most suggestive of: hypotonia. Cerebral palsy. spinal cord injury. neonatal myasthenia gravis.

Cerebral palsy. Poor head control, a persistent Moro reflex, and feeding difficulties in a 6-month-old are suggestive of cerebral palsy. 1. Not smiling, poor head control, a persistent Moro reflect, and gagging and choking while eating are not consistent with hypotonia, spinal cord injury, or neonatal myasthenia gravis. 3. Not smiling, poor head control, a persistent Moro reflect, and gagging and choking while eating are not consistent with hypotonia, spinal cord injury, or neonatal myasthenia gravis. 4. Not smiling, poor head control, a persistent Moro reflect, and gagging and choking while eating are not consistent with hypotonia, spinal cord injury, or neonatal myasthenia gravis.

236. A school nurse is preparing a physical education plan for a child with Down syndrome. Before preparing the plan, the nurse obtains a copy of an x-ray report of the child's: 1. Cervical spine 2. Hands 3. Heart 4. Chest and lungs

Cervical spine Rationale: Children with Down syndrome frequently have instability of the space between the first two cervical vertebrae. They require diagnostic studies (an x-ray of the cervical spine) to determine if this is present before participating in activities that put pressure on the head and neck, which could cause spinal cord compression. Options 2, 3, and 4 are not necessary.

250. A nurse is assisting in preparing a plan of care for a child who will be returning from surgery following the application of a hip spica cast. Which of the following would be the priority in the plan of care for this child on return from the procedure? 1. Elevate the head of the bed. 2. Turn the child onto the right side. 3. Check circulation in the feet. 4. Abduct the hips using pillows.

Check circulation in the feet. Rationale: During the first few hours after a cast is applied, the primary concern is swelling that may cause the cast to produce a tourniquet-like effect and restrict circulation. Therefore circulatory assessment is a priority. Elevating the head of the bed of a child in a hip spica would cause discomfort. Using pillows to abduct the hips is not necessary because a hip spica immobilizes the hip and the knee. Turning the child side to side at least every 2 hours is important because it allows the body cast to dry evenly and prevents complications related to immobility; however, it is not as important as checking circulation.

311. A nurse is monitoring a child with a cast on the forearm for signs of compartment syndrome. The nurse understands that which data collection technique is unlikely to provide information about this complication? 1. Checking the quality of the radial pulse 2. Checking the child's ability to extend the fingers 3. Checking for effectiveness of analgesics administered for pain 4. Checking the child's ability to perform range of motion to the shoulder area of the affected extremity

Checking the child's ability to perform range of motion to the shoulder area of the affected extremity Rationale: Compartment syndrome occurs when swelling causes pressure to rise within a compartment (sheath of inelastic fascia). The increased pressure compromises circulation to the muscles and nerves within the compartment and can result in paralysis and necrosis of tissues. Signs of compartment syndrome include severe pain, often unrelieved by analgesics, and signs of neurovascular impairment. Compartment syndrome is not uncommon in fractures of the forearms; therefore the quality of the radial pulse and the ability to extend the fingers should be assessed. If extension of the fingers produces pain, the health care provider should be notified. Option 4 is unlikely to provide information about compartment syndrome.

187. A nurse is preparing to perform a neurovascular check for tissue perfusion in the child with an arm cast. Which of the following is the priority when performing this procedure? 1. Taking the temperature 2. Taking the blood pressure 3. Checking the apical heart rate 4. Checking the peripheral pulse in the affected arm

Checking the peripheral pulse in the affected arm Rationale: The neurovascular check for tissue perfusion is performed on the toes or fingers distal to an injury or cast and includes checking peripheral pulse, color, capillary refill time, warmth, motion, and sensation. Options 1, 2, and 3 may be components of care, but they are not the priority in this situation.

232. A nurse is caring for a newborn with a diagnosis of spina bifida (myelomeningocele). To monitor for a major symptom associated with this disorder, the nurse: 1. Checks the capillary refill on the nail beds of the upper extremities 2. Tests the urine for blood 3. Palpates the abdomen for masses 4. Checks for responses to painful stimuli from the torso downward

Checks for responses to painful stimuli from the torso downward Rationale: Newborns with spina bifida (myelomeningocele type) demonstrate lack of nerve innervation from below the site of the gibbus (sac containing the meninges and spinal cord). They therefore show diminished or no responses to painful stimuli in the areas below the gibbus. Options 1, 2, and 3 are incorrect because the area above the gibbus is not affected. The capillary refill would be normal. The urine would not have blood present. If the kidneys are affected, proteinuria could be present but this is not generally noted in the newborn period. No masses are present besides the gibbus on the back area, externally protruding from the vertebral deformity.

73. Which of the following types of diarrhea is commonly seen in malabsorption syndromes because the intestine cannot absorb nutrients or electrolytes? Chronic Secretory Acute Intractable diarrhea of infancy

Chronic Chronic diarrhea is commonly seen in malabsorption syndromes. In disorders such as lactose intolerance, the intestine cannot absorb nutrients or electrolytes. Secretory diarrhea is usually caused by bacterial enterotoxins that stimulate fluid and electrolyte secretion. Acute diarrhea is caused by infectious agent in the gastrointestinal tract. Intractable diarrhea of infancy is a syndrome that occurs in the first few months of life and persists longer than 2 weeks with no recognized pathogens.

138. An adolescent diagnosed with thalassemia major (Cooley's anemia) is at risk for which condition?

Chronic hypoxia and iron overload Rationale: increases destruction of red blood cells (RBCs), shortens the life span of RBCs, and causes anemia. The body responds by increasing RBC production, but it can't produce adequate numbers of mature cells. This process results in chronic hypoxia. In addition, children with thalassemia major require multiple transfusions of packed RBCs. The combination of excessive RBC destruction and multiple transfusions deposits excess iron that damages organs and tissues.

318. A nurse has reviewed the health care provider's prescriptions for a child suspected of a diagnosis of neuroblastoma and is preparing to implement diagnostic procedures that will confirm the diagnosis. The nurse prepares to: 1. Collect a 24-hour urine sample. 2. Perform a neurological assessment. 3. Send the child to the radiology department for a chest x-ray. 4. Assist with a bone marrow aspiration.

Collect a 24-hour urine sample Rationale: Neuroblastoma is a solid tumor found only in children. It arises from neural crest cells that develop into the sympathetic nervous system and the adrenal medulla. Typically the tumor infringes on adjacent normal tissue and organs. Neuroblastoma cells may excrete catecholamines and their metabolites. Urine samples will indicate elevated vanillylmandelic acid levels. A bone marrow aspiration will assist in determining marrow involvement. A neurological examination and a chest x-ray may be performed but will not confirm the diagnosis.

120. In teaching a group of nursing students about factors that could lead to the development of urinary tract infections, which critical aspect should the nursing instructor focus on? Concept of urinary stasis Over distention of the bladder Urinary frequency Maintaining proper hydration

Concept of urinary stasis Urinary stasis is the primary factor leading to the occurrence of urinary tract infections as retaining urine serves as an excellent medium for bacterial growth. Over distention of the bladder is a direct result of urinary stasis. Urinary frequency as a clinical symptom rather than an etiologic cause is of concern as is the intervention of maintaining proper hydration to help flush components through the urinary tract system.

89. A young child has recently been fitted with a knee, ankle, and foot orthosis (brace). Care of the skin should include which of the following? Apply lotion or cream to soften the skin. Contact a practitioner or orthotist if skin redness does not disappear. Place padding between the skin and brace if the child experiences a burning sensation under the brace. If a small blister develops, apply rubbing alcohol and place padding between the skin and the brace.

Contact a practitioner or orthotist if skin redness does not disappear. Redness is a sign of skin irritation from the brace. The brace needs to be adjusted to be functional. The skin should not be softened. The brace is specially designed for the child. Padding may alter the alignment of the brace. Rubbing alcohol would be painful. If the brace causes blisters, it needs to be adjusted.

302. A nurse caring for a child with nephrotic syndrome reviews the medication record. The nurse notes that prazosin hydrochloride (Minipress) is prescribed for the child. The nurse determines that this medication has been prescribed to: 1. Reduce proteinuria. 2. Decrease inflammation. 3. Suppress the autoimmune response. 4. Control hypertension.

Control hypertension. Rationale: Prazosin hydrochloride (Minipress) may be used to control hypertension. The child also may be placed on diuretic therapy until protein loss is controlled. Corticosteroids, such as prednisone, may be prescribed to decrease inflammation. Corticosteroids also suppress the autoimmune response and stimulate vascular reabsorption of edema. Cyclophosphamide is an alkylating agent and may be used in maintaining remission.

70. Therapeutic management of nephrotic syndrome includes which of the following? Corticosteroids Long-term diuretics Antihypertensive agents Fluid and salt restrictions

Corticosteroids Most children with nephrotic syndrome respond to corticosteroids, making this group the drug of choice. Corticosteroid therapy is begun as soon as the diagnosis has been determined. Children with nephrotic syndrome usually do not respond to diuretics. Furosemide, in combination with metolazone, is useful for severe edema. Antihypertensive agents are not indicated in the management. Fluids are rarely restricted. The child is placed on a no-added-salt diet.

305. A nursing instructor is observing a nursing student caring for an infant with a diagnosis of bladder exstrophy. The nursing student provides appropriate care to the infant by: 1. Covering the bladder with a dry sterile dressing 2. Covering the bladder with a wet-to-dry dressing 3. Applying sterile water soaks to the bladder mucosa 4. Covering the bladder with a nonadhering plastic wrap

Covering the bladder with a nonadhering plastic wrap Rationale: Care should be taken to protect the exposed bladder tissue from drying while allowing drainage of urine. This is best accomplished by covering the bladder with a nonadhering plastic wrap. The use of wet-to-dry dressings should be avoided because this type of dressing adheres to the mucosa and may damage the delicate tissue when removed. Sterile dressings and dressings soaked in solutions can also dry out and damage the mucosa when removed.

65. Which of the following urine tests of renal function is used to estimate glomerular filtration? pH Creatinine Osmolality Protein level

Creatinine The most useful clinical estimation of glomerular filtration is the clearance of creatinine. The production and secretion of creatinine remain relatively constant from day to day, and its appearance in the urine is determined by the serum level. The pH is a measure of alkalinity, not glomerular filtration. Osmolality is a measure of concentration. The presence of protein is indicative of abnormal glomerular permeability.

101. Which of the following is characterized by a chronic inflammatory process that may involve any part of the gastrointestinal (GI) tract, from mouth to anus? Crohn disease Ulcerative colitis Meckel diverticulum Irritable bowel syndrome

Crohn's disease 1. The chronic inflammatory process of Crohn disease involves any part of the GI tract from the mouth to the anus, but most often affects the terminal ileum. 2. Ulcerative colitis, Meckel diverticulum, and irritable bowel syndrome do not affect the entire GI tract. 3. Ulcerative colitis, Meckel diverticulum, and irritable bowel syndrome do not affect the entire GI tract. 4. Ulcerative colitis, Meckel diverticulum, and irritable bowel syndrome do not affect the entire GI tract.

209. A nurse is checking the capillary refill of a child with a cast applied to the left arm. The nurse compresses the nail bed of a finger and it returns to its original color in 2 seconds. Which action should be taken by the nurse? 1. Document the findings. 2. Notify the registered nurse (RN). 3. Prepare the child for bivalving the cast. 4. Elevate the extremity and recheck the capillary refill immediately.

Document the findings. Rationale: When checking capillary refill, the nurse would expect to note that a compressed nail bed will return to its original color in less than 3 seconds. Options 2, 3, and 4 are unnecessary actions.

116. You are educating nursing student regarding fluid requirements for pediatric patients who present with comorbidities. Increased need for fluid requirements would be consistent with treatment management for which conditions? (Select all that apply.) Congestive Heart Failure (CHF) Diabetic ketoacidosis (DKA) Syndrome of inappropriate diuretic hormone (SIADH) Diabetes Insipidus(DI) Burns

Diabetic ketoacidosis (DKA) Diabetes Insipidus(DI) Burns

284. A 2-year-old child is admitted to the hospital with a diagnosis of nephrotic syndrome. In planning care for this child, which of the following nursing interventions would be of highest priority? 1. Weigh morning and afternoon. 2. Maintain a strict intake and output. 3. Dipstick the urine for protein every 4 hours. 4. Take vital signs with blood pressure every 4 hours.

Dipstick the urine for protein every 4 hours. Rationale: Continuous monitoring of fluid retention and excretion is an important nursing intervention in the care of the child with nephrotic syndrome. Although it is important to maintain a strict intake and output in monitoring fluid retention and excretion, the goal of treatment with this child is to decrease the amount of protein lost in the urine. Because this is the goal, option 3 has the highest priority. Although weight is monitored, it is not necessary to check the weight morning and evening. Taking vital signs with blood pressure is important but is not the priority in this situation.

68. In acute glomerulonephritis, the nurse is aware that an early warning sign of encephalopathy is which of the following? Seizures Psychosis Dizziness Transient loss of vision

Dizziness Acute and severe hypertension can cause the protective autoregulation of cerebral blood flow to fail, leading to hyperperfusion of the brain and cerebral edema. The premonitory signs of encephalopathy are headache, dizziness, abdominal discomfort, and vomiting. Seizures and transient loss of vision are signs that the condition is progressing. Psychosis is not an early warning sign of hypertensive encephalopathy. Seizures and transient loss of vision are signs that the condition is progressing.

264. A nurse is assigned to assist in caring for a newborn with a colostomy that was created during surgical intervention for imperforate anus. When the newborn returns from surgery, the nurse checks the stoma and notes that it is red and edematous. Which of the following is the appropriate nursing intervention? 1. Notify the registered nurse immediately. 2. Document the findings. 3. Apply ice immediately. 4. Elevate the buttocks.

Document the findings. Rationale: A fresh colostomy stoma will be red and edematous but this will decrease with time. The colostomy site will then be pink without evidence of abnormal drainage, swelling, or skin breakdown. The nurse would document these findings because this is a normal expectation. Options 1, 3, and 4 are inappropriate interventions.

297. A female adolescent with type 1 diabetes mellitus will become a member of the school's football cheerleader team. The adolescent excitedly reports to the school nurse to obtain information regarding adjustments needed in the treatment plan for the diabetes. The school nurse would instruct the adolescent to: 1. Eat six graham crackers or drink a cup of orange juice before practice or game time. 2. Eat half the amount of food normally eaten at lunchtime. 3. Take the prescribed insulin one half hour before practice or game time rather than in the morning. 4. Take two times the amount of prescribed insulin on practice and game days.

Eat six graham crackers or drink a cup of orange juice before practice or game time. Rationale: An extra snack of 15 to 30 g of carbohydrate eaten before activities such as cheerleader practice will prevent hypoglycemia. Six graham crackers or a cup of orange juice will provide 15 to 30 g of carbohydrate. The adolescent should not be instructed to adjust the amount or time of insulin administration. Meal amounts should not be decreased.

255. An infant returns to the nursing unit following surgery for an esophageal atresia with tracheoesophageal fistula (TEF). The infant is receiving intravenous (IV) fluids, and a gastrostomy tube is in place. The nurse who is assisting in caring for the infant will ensure that the gastrostomy tube is: 1. Placed to gravity 2. Attached to low suction 3. Taped to the bed linens 4. Elevated

Elevated Rationale: In the immediate postoperative period, the gastrostomy tube is elevated, allowing gastric contents to pass to the small intestine and air to escape. This promotes comfort and decreases the risk of leakage at the anastomosis. Options 1, 2, and 3 are incorrect.

239. A diagnostic workup is performed on a 1-year-old child suspected of a diagnosis of neuroblastoma. Which finding specifically associated with this type of tumor should the nurse expect to find documented in the child's record? 1. Elevated vanillylmandelic acid (VMA) levels in the urine 2. The presence of blast cells in the bone marrow 3. Projectile vomiting occurring often in the morning 4. Positive Babinski's sign

Elevated vanillylmandelic acid (VMA) levels in the urine Rationale: Neuroblastoma is a solid tumor found only in children. It arises from neural crest cells that develop into the sympathetic nervous system and the adrenal medulla. Typically the tumor infringes on adjacent normal tissue and organs. Neuroblastoma cells may excrete catecholamines and their metabolites. Urine samples will indicate elevated VMA levels. The presence of blast cells in the bone marrow occurs in leukemia. Projectile vomiting occurring most often in the morning and a positive Babinski's sign are clinical manifestations of a brain tumor.

144. The physician suspects tracheoesophageal fistula in a 1-day-old neonate. Which nursing intervention is most appropriate for this child?

Elevating the neonate's head and giving nothing by mouth Rationale: because of the risk for aspiration

215. A nurse is assisting in developing a plan of care for a patient diagnosed with acute glomerulonephritis. The nurse includes which intervention in the plan of care? 1. Encourage limited activity and provide safety measures. 2. Force intake of oral fluids to prevent hypovolemic shock. 3. Catheterize the child to strictly monitor intake and output. 4. Encourage classmates to visit and to keep the child informed of school events.

Encourage limited activity and provide safety measures. Rationale: Activity is limited and most children, because of fatigue, voluntarily restrict their activities during the active phase of the disease. Catheterization may cause a risk of infection. Fluids should not be forced. Visitors should be limited to allow for adequate rest.

196. A nurse is assisting in preparing a plan of care for a 4-year-old child hospitalized with nephrotic syndrome. Which dietary intervention is most appropriate for this child? 1. Provide a high-salt diet. 2. Provide a high-protein diet. 3. Discourage visitors at mealtimes. 4. Encourage the child to eat in the playroom.

Encourage the child to eat in the playroom. Rationale: Mealtimes should center on pleasurable socialization. The child should be encouraged to eat meals with other children on the unit. A diet that is normal in protein with a sodium restriction is normally prescribed for a child with nephrotic syndrome. Parents or other family members should be encouraged to be present at mealtimes with a hospitalized child.

104. An infant with short-bowel syndrome will be discharged home on total parenteral nutrition (TPN) and gastrostomy feedings. Nursing care should include which of the following? Prepare family for impending death. Teach family how to calculate caloric needs. Ensure that family can identify signs of central venous catheter infections. Secure TPN and gastrostomy tubing under diaper to lessen risk of dislodgment.

Ensure that family can identify signs of central venous catheter infections. During TPN therapy care must be taken to minimize the risk of complications related to the central venous access device, such as catheter infections, occlusions, or accidental removal. This is an important part of family teaching. 1. The prognosis for patients with short-bowel syndrome depends in part on the length of residual small intestine. It has improved with advances in TPN. 2. Although parents need to be taught about nutritional needs, the caloric needs and prescribed TPN and rate are the responsibility of the health care team. 4. The tubes should not be placed under the diapers because of the risk of infection.

267. A 3-year-old child is seen in the health care clinic, and a diagnosis of encopresis is made. The nurse reviews the record, expecting to note which of the following that is a sign of this disorder? 1. Nausea and vomiting 2. Diarrhea 3. Evidence of soiled clothing 4. Malaise anorexia

Evidence of soiled clothing Rationale: Encopresis is defined as fecal incontinence and is a major concern if the child is constipated. Signs include evidence of soiling clothing, scratching, or rubbing the anal area because of irritation, fecal odor without apparent awareness by the child, and social withdrawal

235. A nurse assists in monitoring for early signs of meningitis in a child and assists with attempting to elicit Kernig's sign. The appropriate procedure to elicit Kernig's sign is to: 1. Bend the head toward the knees and hips and check for pain. 2. Tap the facial nerve and check for spasm. 3. Compress the upper arm and check for tetany. 4. Extend the leg and knee and check for pain

Extend the leg and knee and check for pain. Rationale: Kernig's sign is pain that occurs with extension of the leg and knee. Brudzinski's sign occurs when flexion of the head causes flexion of the hips and knees. Chvostek's sign, seen in tetany, is a spasm of the facial muscles elicited by tapping the facial nerve in the region of the parotid gland. Trousseau's sign is a sign for tetany in which carpal spasm can be elicited by compressing the upper arm and causing ischemia to the nerves distally.

217. A 1-year-old child with hypospadias is scheduled for surgery to correct this condition. A nurse is asked to assist in preparing a plan of care for this child and makes suggestions, knowing that this surgery is taking place at a time when: 1. Fears of separation and mutilation are present 2. Sibling rivalry will cause regression to occur 3. Embarrassment of voiding irregularities is common 4. Concern over size and function of the penis is present

Fears of separation and mutilation are present Rationale: At the age of 1 year, a child's fears of separation and mutilation are present because the child is facing the developmental task of trusting others. As the child gets older, fears about virility and reproductive ability may surface. The question does not provide enough data to determine that siblings exist. Options 3 and 4 may be issues if the child were older.

277. A nurse is caring for an infant with a diagnosis of Hirschsprung's disease. The nurse should check for which clinical findings that are consistent with Hirschsprung's disease? Select all that apply. 1. Fever 2. Constipation 3. Failure to thrive 4. Intolerance to wheat 5. Abdominal distention 6. Explosive, watery diarrhea

Fever Constipation Failure to thrive Abdominal distention Explosive, watery diarrhea Rationale: Clinical manifestations of Hirschsprung's disease during infancy include failure to thrive, constipation, abdominal distention, episodes of diarrhea and vomiting, signs of enterocolitis, explosive and watery diarrhea, and fever. The infant appears significantly ill. Intolerance to wheat occurs in celiac disease

277. A nurse is caring for an infant with a diagnosis of Hirschsprung's disease. The nurse should check for which clinical findings that are consistent with Hirschsprung's disease? Select all that apply. 1. Fever 2. Constipation 3. Failure to thrive 4. Intolerance to wheat 5. Abdominal distention 6. Explosive, watery diarrhea

Fever Constipation Failure to thrive Abdominal distention Explosive, watery diarrhea Rationale: Clinical manifestations of Hirschsprung's disease during infancy include failure to thrive, constipation, abdominal distention, episodes of diarrhea and vomiting, signs of enterocolitis, explosive and watery diarrhea, and fever. The infant appears significantly ill. Intolerance to wheat occurs in celiac disease.

112. In performing a work up for a school aged child who reports frequent abdominal pain symptoms, what information would be critical to collect in order to make an accurate clinical diagnosis? Find out the duration, onset and quality characteristics of the symptoms. Ask the child's parents for detailed information. Find out if the child has any food allergies or food intolerances. Take and document vital signs to establish a clinical baseline.

Find out the duration, onset and quality characteristics of the symptoms. School-age children typically relate recurrent abdominal pain. As such it is critical to obtain factors related to the pain characteristics, onset, duration and symptoms. This will help to correlate with potential organic and non-organic causes. Asking the parents' for detailed information may be needed but not at present to determine what type of pain the child is experiencing. Although it is important to denoted food allergies and food intolerance, the primary concern is abdominal pain presentation. Similarly, vital signs should be recorded but it is not the most critical piece of data that must be collected.

158. A nurse reviews the record of a 3-week-old infant and notes that the health care provider has documented a diagnosis of suspected Hirschsprung's disease. The nurse understands that which of the following symptoms led the mother to seek health care for the infant? 1. Diarrhea 2. Projectile vomiting 3. The regurgitation of feedings 4. Foul-smelling, ribbon-like stools

Foul-smelling, ribbon-like stools Rationale: Chronic constipation that begins during the first month of life and that results in foul-smelling, ribbon-like or pellet-like stools is a clinical manifestation of Hirschsprung's disease. The delayed passage or absence of meconium stool during the neonatal period is a characteristic sign. Bowel obstruction (especially during the neonatal period), abdominal pain and distention, and failure to thrive are also clinical manifestations. Options 1, 2, and 3 are incorrect.

240. A nurse is administering medications to a 6-year-old child with nephrotic syndrome. To reduce proteinuria, the nurse should expect that which medication would be prescribed? 1. Enalapril (Vasotec) 2. Furosemide (Lasix) 3. Prednisone 4. Cyclophosphamide

Furosemide (Lasix) Rationale: The child is usually placed on diuretic therapy with furosemide (Lasix) until protein loss is controlled. Enalapril is most commonly used to control hypertension. Corticosteroids, such as prednisone, may be prescribed to decrease inflammation. Corticosteroids also suppress the autoimmune response and stimulate vascular reabsorption of edema. Cyclophosphamide is an alkylating agent and may be used in maintaining remission.

271. A newborn infant is diagnosed with gastroesophageal reflux (GER). The mother of the infant asks the nurse to explain the diagnosis. The nurse plans to base the response on which description of this disorder? 1. Gastric contents regurgitate back into the esophagus. 2. The esophagus terminates before it reaches the stomach. 3. Abdominal contents herniate through an opening of the diaphragm 4. A portion of the stomach protrudes through the esophageal hiatus of the diaphragm.

Gastric contents regurgitate back into the esophagus. Rationale: GER is regurgitation of gastric contents back into the esophagus. Option 2 describes esophageal atresia. Option 3 describes a congenital diaphragmatic hernia. Option 4 describes a hiatal hernia.

111. What clinical manifestations would the nurse expect to find in a newborn who has developed necrotizing enterocolitis (NEC)? Hyperthermia Gastric residual and melena The passage of ribbon-like stools Projectile vomiting

Gastric residual and melena The most prominent signs of NEC are abdominal distention, gastric residuals, and blood in the stools (melena). NEC resembles septicemia; the newborn may "not look well," in addition to having nonspecific signs such as lethargy, poor feeding, hypotension, hypothermia, bile-stained vomitus, and oliguria. The newborn with NEC is more likely to be seen with hypothermia, not hyperthermia. The passage of ribbon-like stools is seen in newborns and infants born with Hirschsprung disease. Projectile vomiting is seen in newborns and infants with pyloric stenosis.

176. A nurse is assisting with performing admission data collection on a 2-year-old child who has been diagnosed with nephrotic syndrome. The nurse collects data knowing that a common characteristic associated with nephrotic syndrome is: 1. Hypotension 2. Generalized edema 3. Increased urinary output 4. Frank, bright red blood in the urine

Generalized edema Rationale: Nephrotic syndrome is defined as massive proteinuria, hypoalbuminemia, and edema. The urine is dark, foamy, and frothy, but microscopic hematuria may be present. Frank, bright red blood in the urine does not occur. Urine output is decreased, and the blood pressure is normal or slightly decreased.

298. An adolescent with diabetes mellitus is attending gym class and suddenly becomes flushed and complains of dizziness and a headache. The gym teacher quickly takes the adolescent to the school nurse's office. The nurse obtains a blood glucose level, and the results indicate a level of 65 mg/dL. The appropriate initial nursing intervention is to: 1. Call the child's mother for permission to treat the child. 2. Call the school health care provider immediately. 3. Let the child rest until the blood glucose has an opportunity to rise. 4. Give the child 6 oz of a regular cola drink.

Give the child 6 oz of a regular cola drink. Rationale: A blood glucose level below 70 mg/dL indicates hypoglycemia. The child is participating in an activity that requires more energy than that of the normal routine at school. Insulin and food requirements change with situations that require more energy. When signs of hypoglycemia occur, the child needs an immediate source of glucose. Options 1, 2, and 3 do not address the hypoglycemic state immediately and delay required treatment.

259. A breast-feeding mother of an infant with lactose intolerance asks the nurse about dietary measures. Which of the following foods would the nurse instruct the mother to avoid? 1. Hard cheeses 2. Green, leafy vegetables 3. Dried beans 4. Egg yolk

Hard cheeses Rationale: Breast-feeding mothers of an infant with lactose intolerance need to be encouraged to limit dairy products. Cheese is a dairy product. Alternative calcium sources include egg yolk; green, leafy vegetables; dried beans; cauliflower; and molasses.

79. The school nurse is called to the cafeteria because a child "has eaten something he is allergic to." The child is in severe respiratory distress. The nurse should do which of the following first? Determine what the child has eaten. Administer diphenhydramine (Benadryl). Move the child to the nurse's office or hallway. Have someone call for an ambulance or paramedic rescue squad.

Have someone call for an ambulance or paramedic rescue squad. Because the child is in severe respiratory distress, the nurse should remain with the child while someone else calls for the rescue squad. Because severe respiratory distress is occurring, treatment of the response is indicated first. The cause of the response can be determined later. Diphenhydramine will not be effective for this type of allergic reaction. The child should not be moved unless the child is in a place that puts the child at greater hazard.

279. A child has epistaxis. The nurse understands that an appropriate treatment for epistaxis is which of the following? 1. Have the child sit up and lean forward. 2. Have the child assume a supine position. 3. Have the child sit up and tilt the head backward. 4. Apply continuous pressure to the nose for at least 3 minutes.

Have the child sit up and lean forward. Rationale: Correct treatment for epistaxis (a nosebleed) involves having a client sit up and lean forward. Therefore options 2 and 3 are incorrect. Continuous pressure should be applied to the nose for at least 10 minutes.

122. Disorders of sexual development (DSD) present unique challenges to both the patient, family members and health care providers. In preparing for health promotion materials to be used in a teaching session for health care providers on this subject, which information should be included? Health care providers should understand their own feelings and implicit bias relative to DSD. There is no need for referrals to specialized treatment centers as DSD can be handled in any hospital setting. The primary concerns rest with the physical presentations rather than psychological/psychosocial concerns.

Health care providers should understand their own feelings and implicit bias relative to DSD. Most of the issues regarding DSD resolve as the individual ages and becomes more mature. In order to treat patients, all health care providers must have an understanding of their own individual feelings and implicit bias. DSD is a complex disease process in which psychological/psychosocial feelings are equally if not more important than physical presentations. Referrals to specialized care settings and use of an interdisciplinary support team should be encouraged. Growth and development alone do not obliterate the effects of DSD.

149. Which of the following actions can help stop child abuse and neglect? a. Helping a stressed-out parent by baby-sitting, making a meal for their family, or lending an understanding ear b. Learning the signs and symptoms of child abuse so you can recognize them when you see the "red flags" c. Reporting known or suspected child abuse to the police or local child protective services agency d. All of the above

Helping a stressed out parent by babysitting, making a meal for their family, or lending an understanding ear Learning the signs and symptoms of child abuse so you can recognize them when you see the "red flags" Reporting known or suspected child abuse to the police or local child protective services agency

227. A nurse is reviewing the health record of an infant with a diagnosis of gastroesophageal reflux. Which clinical manifestation of this disorder would the nurse expect to note documented in the record? 1. Excessive oral secretions 2. Bowel sounds heard over the chest 3. Hiccupping and spitting up after a meal 4. Coughing, wheezing, and short periods of apnea

Hiccupping and spitting up after a meal Rationale: Clinical manifestations of all types of gastroesophageal reflux include vomiting (spitting up) after a meal, hiccupping, and recurrent otitis media related to pooled secretions in the nasopharynx during sleep. Option 1 is a clinical manifestation of esophageal atresia and tracheoesophageal fistula. Option 2 is a clinical manifestation of congenital diaphragmatic hernia. Option 4 is a clinical manifestation of hiatal hernia.

54. What is frequently associated with infant botulism? Contaminated soil Honey and corn syrup Commercial infant cereals Improperly sterilized bottles

Honey and corn syrup These substances are not usually associated with infants who have become affected. Unlike adult botulism, infant botulism is caused by ingesting spores of C. botulinum and the resultant release of toxin. The bacterium has been found in honey and corn syrup that was fed to affected infants. These substances are not usually associated with infants who have become affected. These substances are not usually associated with infants who have become affected.

102. Pyloric stenosis can best be described as which of the following? Dilation of pylorus Hypertrophy of pyloric muscle Hypotonicity of pyloric muscle Reduction of tone in the pyloric muscle

Hypertrophy of pyloric muscle 2. Hypertrophic pyloric stenosis occurs when the circumferential muscle of the pyloric sphincter becomes thickened, resulting in elongation and narrowing of the pyloric channel. 1. Dilation of pylorus, hypotonicity of pyloric muscle, and reduction of tone in the pyloric muscle are not the definition of pyloric stenosis. 3. Dilation of pylorus, hypotonicity of pyloric muscle, and reduction of tone in the pyloric muscle are not the definition of pyloric stenosis. 4. Dilation of pylorus, hypotonicity of pyloric muscle, and reduction of tone in the pyloric muscle are not the definition of pyloric stenosis.

28. Which laboratory finding, in conjunction with the presenting symptoms, indicates nephrosis? 1. Hypoalbuminemia 2. Low specific gravity 3. Decreased hemoglobin 4. Decreased hematocrit

Hypoalbuminemia Hypoalbuminemia is a result of the large amounts of protein that leak through the glomerular membrane into urine. Specific gravity is increased because of the large amount of protein. These measures would be elevated secondary to the hypovolemia. These measures would be elevated secondary to the hypovolemia.

314. A nurse is providing instructions to the mother of a child with juvenile idiopathic arthritis regarding measures to take if a painful exacerbation of the disease occurs. Which statement by the mother indicates the need for further instruction? 1. "The full range-of-motion (ROM) exercises must be performed every day, even during the exacerbations." 2. "Hot or cold packs will assist in reducing discomfort." 3. "The painful joint should be splinted and positioned in a neutral position." 4. "I should have my child perform simple isometric exercises during exacerbations."

I should have my child perform simple isometric exercises during exacerbations Rationale: During painful episodes, hot or cold packs and splinting and positioning the affected joint in a neutral position help reduce the pain. Full ROM exercises will cause significant pain during exacerbation and should be avoided during this time. Although resting the extremity is appropriate, it is important to begin simple isometric or tensing exercises as soon as the child is able. These exercises do not involve joint movement.

1. What factor predisposes an infant to fluid imbalances? Decreased surface area Lower metabolic rate Immature kidney functioning Decreased daily exchange of extracellular fluid

Immature kidney functioning The infant has a proportionately greater body surface area, which allows for greater insensible water loss. The infant has a higher metabolic rate. The infant's kidneys are unable to concentrate or dilute urine, conserve or excrete sodium, or acidify urine. This is an increased amount of extracellular fluid in the infant. Approximately 60% of fluid loss is from extracellular space.

121. Parents of a newborn bring their male son to the emergency room. The infant appears fretful and the parents state that he has not voided in several hours. Inspection of the penis reveals edema and the nurse is unable to retract the foreskin. Based on this assessment, what would the nurse anticipate as the priority action? Perform an ultrasound to determine if there is urinary retention. Inform the ER physician of the patient's condition. Ask the parents specifically how long the infant has not voided. Continue to monitor the patient in the ER setting.

Inform the ER physician of the patient's condition. If the nurse cannot retract the foreskin and there is associated edema, paraphimosis is suspected which requires prompt medical evaluation. Ultrasound determination of bladder status is not the priority action. Even though having more precise information about the length of time that the infant has not voided, the priority is to initiate medical action.

172. A nurse is assigned to care for a child who is scheduled for an appendectomy. Which prescriptions does the nurse anticipate will be prescribed? Select all that apply. 1. Administer a Fleet enema. 2. Initiate an intravenous line. 3. Maintain nothing-by-mouth status. 4. Administer intravenous antibiotics. 5. Administer preoperative medications. 6. Place a heating pad on the abdomen to decrease pain.

Initiate an intravenous line. Maintain nothing-by-mouth status. Administer intravenous antibiotics. Administer preoperative medications. Rationale: During the preoperative period, enemas or laxatives should not be administered. In addition, heat should not be applied to the abdomen. Any of these interventions can cause the rupture of the appendix and resultant peritonitis. Intravenous fluids would be started, and the child should receive nothing by mouth while awaiting surgery. Antibiotics are usually administered because of the risk of perforation. Preoperative medications are administered as prescribed.

308. A 4-year-old child is being transported to the trauma center from a local community hospital for treatment of a burn injury that is estimated as covering over 40% of the body. The burns are partial- and full-thickness burns. The nurse is asked to prepare for the arrival of the child and gathers supplies anticipating that which of the following will be prescribed initially? 1. Insertion of a nasogastric tube 2. Insertion of a Foley catheter 3. Administration of an anesthetic agent for sedation 4. Application of an antimicrobial agent to the burns

Insertion of a Foley catheter Rationale: A Foley catheter is inserted into the child's bladder so that urine output can be measured accurately on an hourly basis. Although pain medication may be required, the child would not receive an anesthetic agent and should not be sedated. The burn wounds would be cleansed and treated after assessment, but this would not be the initial action. Intravenous fluids are administered at a rate sufficient to keep the child's urine output at 1 mL/kg of body weight per hour, thus reflecting adequate tissue perfusion. A nasogastric tube may or may not be required but would not be the priority intervention.

319. A nurse is asked to prepare for the admission of a child to the pediatric unit with a diagnosis of Wilms' tumor. The nurse assists in developing a plan of care for the child and suggests including which of the following in the plan of care? 1. Palpate the abdomen for an increase in the size of the tumor every 8 hours. 2. Inspect the urine for the presence of hematuria at each voiding. 3. Monitor the temperature for hypothermia. 4. Monitor the blood pressure for hypotension.

Inspect the urine for the presence of hematuria at each voiding. Rationale: If Wilms' tumor is suspected, the tumor mass should not be palpated. Excessive manipulation can cause seeding of the tumor and cause spread of the cancerous cells. Fever (not hypothermia), hematuria, and hypertension (not hypotension) are clinical manifestations associated with Wilms' tumor.

127. A nurse is taking care of a pediatric patient whose health record indicates E on the American Spinal Injury Association Impairment Scale. What does this designation mean? Sensory function present but no motor function No motor or sensory function Motor function is preserved below the neurologic level Intact motor and sensory function

Intact motor and sensory function Designation of E notation on the American Spinal Injury Association Impairment Scale indicates that motor and sensory function are normal. Sensory but no motor function is noted as B-incomplete. No motor or sensory function is noted as A-incomplete. Motor function is preserved below the neurologic level is noted as C-incomplete.

269. A child is diagnosed with intussusception. The nurse collects data on the child, knowing that which of the following is a characteristic of this disorder? 1. The presence of fecal incontinence 2. Incomplete development of the anus 3. The infrequent and difficult passage of dry stools 4. Invagination of a section of the intestine into the distal bowel

Invagination of a section of the intestine into the distal bowel Rationale: Intussusception is an invagination of a section of the intestine into the distal bowel. It is the most common cause of bowel obstruction in children age 3 months to 6 years. Option 1 describes encopresis. Option 2 describes imperforate anus, and this disorder is diagnosed in the neonatal period. Option 3 describes constipation. Constipation can affect any child at any time, although it peaks at ages 2 to 3 years. Encopresis generally affects preschool and school-age children

200. A nursing student is asked to discuss the topic of clubfoot at a clinical conference. The student plans to tell the group that clubfoot: 1. Is a congenital anomaly 2. Always occurs bilaterally 3. Affects girls more often than boys 4. Is a rare deformity of the skeletal system

Is a congenital anomaly Rationale: Clubfoot, one of the most common deformities of the skeletal system, is a congenital anomaly characterized by a foot that has been twisted inward or outward. The condition generally affects both feet, and boys are affected twice as often as girls.

213. A child with a fractured femur is placed in Buck's skin traction and the nurse is planning care for the client. Which information about this type of traction is correct? 1. Requires frequent pin care 2. Places the child at risk for infection 3. Uses skeletal traction and weights to provide a counterforce 4. Is a type of skin traction that pulls the hip and leg into extension

Is a type of skin traction that pulls the hip and leg into extension Rationale: Buck's skin traction is a type of skin traction used in fractures of the femur and in hip and knee contractures. It pulls the hip and leg into extension. Countertraction is applied by the child's body. Options 1, 2, and 3 describe skeletal traction.

220. A child is seen in the clinic, and the primary health care provider documents a diagnosis of primary nocturnal enuresis. The mother asks the nurse about the diagnosis. The nurse bases the response on the fact that primary nocturnal enuresis: 1. Does not respond to treatment 2. Is caused by a psychiatric problem 3. Requires surgical intervention to improve the problem 4. Is common and most children will outgrow bed-wetting without therapeutic intervention

Is common and most children will outgrow bed-wetting without therapeutic intervention Rationale: Primary nocturnal enuresis is bedwetting and is described as occurring in a child that has never been dry at night for extended periods. It is common in children, most of whom will outgrow bedwetting without therapeutic intervention. The child is not able to sense a full bladder and does not awaken to void. The child may have delayed maturation of the central nervous system (CNS). It is not caused by a psychiatric problem. Behavioral conditioning with use of alarms has been used for treatment in the older child with nocturnal enuresis. A device that contains a moisture-sensitive alarm is worn on the child's pajamas. As the child starts to void, the alarm goes off, awakening the child. The alarm system may need to be used consistently over 15 weeks for resolution.

262. A mother of an infant diagnosed with Hirschsprung's disease asks the nurse about the disorder. The nurse plans to base the response on which of the following? 1. It is a congenital aganglionosis or megacolon. 2. It is a complete small intestinal obstruction. 3. It is a condition that causes the pyloric valve to remain open. 4. It is a severe inflammation of the gastrointestinal tract.

It is a congenital aganglionosis or megacolon. Rationale: Hirschsprung's disease, also known as "congenital aganglionosis" or "megacolon," is the result of an absence of ganglion cells in the rectum and to varying degrees upward in the colon. Options 2, 3, and 4 are incorrect.

51. Which statement BEST describes pseudohypertrophic (Duchenne) muscular dystrophy? It is inherited as an autosomal dominant disorder. It is characterized by weakness of proximal muscles of both pelvic and shoulder girdles. It is characterized by muscle weakness usually beginning about 3 years old. Onset occurs in later childhood and adolescence.

It is characterized by muscle weakness usually beginning about 3 years old. It is inherited as an X-linked recessive trait. The first weakness is usually noted in walking. Then a progressive involvement of other muscle groups occurs. Usually children with Duchenne muscular dystrophy reach the early developmental milestones; the muscular weakness is usually observed in the third year of life. Onset usually develops in the third year of life.

193. A nurse prepares a list of home care instructions for the parents of a child who has a plaster cast applied to the left forearm. Choose the instructions that would be included on the list. Select all that apply. 1. Use the fingertips to lift the cast while it is drying. 2. Keep small toys and sharp objects away from the cast. 3. Use a padded ruler or another padded object to scratch the skin under the cast if it itches. 4. Place a heating pad on the lower end of the cast and over the fingers if the fingers feel cold. 5. Contact the health care provider if the child complains of numbness or tingling in the extremity. 6. Elevate the extremity on pillows for the first 24 to 48 hours after casting to prevent swelling.

Keep small toys and sharp objects away from the cast. Contact the health care provider if the child complains of numbness or tingling in the extremity. Elevate the extremity on pillows for the first 24 to 48 hours after casting to prevent swelling. Rationale: While the cast is drying, the palms of the hands are used to lift the cast. If the fingertips are used, indentations in the cast could occur and cause constant pressure on the underlying skin. Small toys and sharp objects are kept away from the cast, and no objects (including padded objects) are placed inside of the cast because of the risk of altered skin integrity. A heating pad is not applied to the cast or fingers. Cold fingers could indicate neurovascular impairment, and the HCP should be notified. The extremity is elevated to prevent swelling, and the HCP is notified immediately if any signs of neurovascular impairment develop.

152. Which of the following represents a primary characteristic of autism? 1. Normal social play 2. Consistent imitation of others' actions 3. Lack of social interaction and awareness 4. Normal verbal and nonverbal communication

Lack of social interaction and awareness Rationale: Autism is a severe form of an autism spectrum disorder. A primary characteristic is a lack of social interaction and awareness. Social behaviors in autism include a lack of or an abnormal imitation of others' actions and a lack of or abnormal social play. Additional characteristics include a lack of or impaired verbal communication and markedly abnormal nonverbal communication.

124. A newborn has been diagnosed with spinal bifida. Which allergy documentation should the nurse include in a plan of care for this child? Penicillin Cloth tape Latex Augmentin

Latex Due to the likelihood of surgeries and procedures, the child with spinal bifida is at increased risk of developing allergic reactions to latex. The development of drug allergies would not typically occur unless there was some underlying autoimmune response. Similarly, the use of cloth tape is not associated with an allergic profile.

183. The nurse assists in planning care for a child who sustained a burn injury based on which of the following accurate statements? 1. Scarring is not as severe in a child as in an adult. 2. Children are at a lower risk of infection than adults because of their strong immune systems. 3. Lower burn temperatures and shorter exposure to heat can cause a more severe burn in a child than an adult because a child's skin is thinner. 4. Infants and children are at decreased risk for protein and calorie deficiency because they have smaller muscle mass and less body fat than adults.

Lower burn temperatures and shorter exposure to heat can cause a more severe burn in a child than an adult because a child's skin is thinner. Rationale: Lower burn temperatures and shorter exposure to heat can cause a more severe burn in a child than an adult because a child's skin is thinner. Scarring is more severe in a child; additionally, disturbed body image will be a distinct issue for a child or adolescent, especially as growth continues. An immature immune system presents an increased risk of infection for infants and young children. Infants and children are at increased risk for protein and calorie deficiency because they have smaller muscle mass and less body fat than adults.

107. Which of the following clinical manifestations in an infant would be suggestive of spinal muscular atrophy (Werdnig-Hoffmann disease)? Hypertonicity Lying in the frog position Hyperactive deep tendon reflexes Motor deficits on one side of body

Lying in the frog position The infant lies in the frog position with the legs externally rotated abducted and flexed at knees. 1. The child has hypotonia and inactivity as the most prominent features. 3. The deep tendon reflexes are absent. 4. The motor deficits are bilateral.

317. A nurse is collecting data from a child suspected of having juvenile idiopathic arthritis (JIA). Which findings would the nurse expect to note if JIA were present? Select all that apply. 1. Malaise, fatigue, and lethargy 2. Painful, stiff, and swollen joints 3. Limited range of motion of the joints 4. Stiffness that develops later in the day 5. Cool temperature of the skin over the affected joints 6. History of late afternoon temperature, with temperature spiking up to 105° F

Malaise, fatigue, and lethargy Painful, stiff, and swollen joints Limited range of motion of the joints History of late afternoon temperature, with temperature spiking up to 105° F Rationale: Clinical manifestations associated with JIA include intermittent joint pain that lasts longer than 6 weeks and painful, stiff, and swollen joints that are warm to the touch, with limited range of motion. The child will complain of morning stiffness and may protect the affected joint or refuse to walk. Systemic symptoms include malaise, fatigue and lethargy, anorexia, weight loss, and growth problems. A history of a late afternoon fever with temperature spiking up to 105° F will also be part of the clinical manifestations.

204. The primary goal to be included in the plan of care for a child who has cerebral palsy is to: 1. Eliminate the cause of the disease. 2. Improve muscle control and coordination. 3. Prevent the occurrence of emotional disturbances. 4. Maximize the child's assets and minimize the limitations.

Maximize the child's assets and minimize the limitations. Rationale: The goal of managing the child with cerebral palsy is early recognition and intervention to maximize the child's abilities. The cause of the disease cannot be eliminated. It is best to minimize emotional disturbances, if possible, but not to prevent them because it is healthy for the child to express emotions. Improvement of muscle control and coordination is a component of the plan, but the primary goal is to maximize the child's assets and minimize the limitations caused by the disease.

272. A nurse is reviewing the laboratory results of an infant suspected of having hypertrophic pyloric stenosis. Which of the following would the nurse expect to note in this infant? 1. Metabolic acidosis 2. Metabolic alkalosis 3. Respiratory acidosis 4. Respiratory alkalosis

Metabolic alkalosis Rationale: Laboratory findings in an infant with hypertrophic pyloric stenosis include metabolic alkalosis as a result of the vomiting (depletes acid) that occurs in this disorder. Additional findings include decreased serum potassium and sodium levels, increased pH and bicarbonate, and decreased chloride level.

287. A 3-year-old child is brought to the emergency department. The mother states that the child has had flulike symptoms with vomiting and diarrhea for the past 2 days. On data collection the nurse finds that the child's heart rate is slightly elevated and the blood pressure is normal. The child is irritable and crying only a few tears. The mother states that the child's weight before the illness was 33 pounds. The nurse finds the current weight to be 31 pounds. The nurse correctly interprets this as what level of dehydration? 1. Mild dehydration 2. Severe dehydration 3. Very mild dehydration 4. Moderate dehydration

Moderate dehydration Rationale: Moderate dehydration demonstrates itself with a weight loss in children of 6% to 8% of weight. Mild dehydration would not present with these symptoms. In severe dehydration, additional findings would include lethargy and listlessness. The symptoms listed are all characteristics of moderate dehydration. Very mild dehydration is not a term used to describe dehydration.

233. A nurse is caring for a newborn with spina bifida (myelomeningocele type) who is scheduled for the removal of the gibbus (sac on the back filled with cerebrospinal fluid, meninges, and some of the spinal cord). In the preoperative period, the priority nursing action is to monitor: 1. Blood pressure 2. Moisture of the normal saline dressing on the gibbous area. 3. Specific gravity of the urine 4. Anterior fontanel for depression

Moisture of the normal saline dressing on the gibbous area. Rationale: The newborn is at risk for infection before closure of the gibbus. A sterile normal saline dressing is placed over the gibbus to maintain moisture of the gibbus and its contents. This prevents tearing or breakdown of the skin integrity at the site. Blood pressure is difficult to determine during the newborn period and is not the best indicator of infection. Urine concentration is not well developed in the newborn stage of development. Depression of the anterior fontanel is a sign of dehydration. With spina bifida, an increase in intracranial pressure is more of a priority. A complication of spina bifida would demonstrate a bulging or taut anterior fontanel.

248. A 3-year-old child has returned to his room following a tonsillectomy. Which assessment finding needs immediate notification of the registered nurse? 1. Pulse rate 90, respirations 24 per minute 2. Nasal flaring and rib retractions 3. Drooling slightly blood-tinged saliva 4. Refusal to take sips of his favorite soda

Nasal flaring and rib retractions Rationale: Nasal flaring and rib retractions are signs of respiratory distress, a major concern following a tonsillectomy. These signs require immediate notification. The vital signs are normal for a 3-year-old child. Drooling slightly blood-tinged saliva and refusal to take sips of liquids are common after a tonsillectomy.

203. A nurse is monitoring a 7-year-old child who sustained a head injury in a motor vehicle accident for signs of increased intracranial pressure (ICP). The nurse assesses the child frequently for which early sign of increased ICP? 1. Nausea 2. Papilledema 3. Decerebrate posturing 4. Alterations in pupil size

Nausea Rationale: Nausea is an early sign of increased ICP. Late signs of increased ICP include a significant decrease in level of consciousness, Cushing's triad (increased systolic blood pressure and widened pulse pressure, bradycardia, and irregular respirations), and fixed and dilated pupils. Other late signs include decreased motor response to command, decreased sensory response to painful stimuli, posturing, Cheyne-Stokes respirations, and papilledema.

188. A nurse is performing a neurovascular check on a child with a cast applied to the lower leg. The child complains of tingling in the toes distal to the fracture site. Which action should be taken by the nurse? 1. Elevate the extremity. 2. Document the findings. 3. Notify the health care provider (HCP). 4. Ambulate the child with crutches.

Notify the health care provider (HCP). Rationale: Reduced sensation to touch or complaints of numbness or tingling at a site distal to the fracture may indicate poor tissue perfusion. This finding should be reported to the registered nurse or HCP. Options 1, 2, and 4 are inappropriate and would delay the required and immediate interventions.

191. A nurse is assigned to care for a child after a spinal fusion for the treatment of scoliosis. The child complains of abdominal discomfort and begins to have episodes of vomiting. On data collection, the nurse notes abdominal distention. Which action should the nurse take? 1. Administer an antiemetic. 2. Increase the intravenous fluids. 3. Notify the registered nurse (RN). 4. Place the child in a side-lying Sims' position.

Notify the registered nurse (RN). Rationale: A complication after the surgical treatment of scoliosis is superior mesenteric artery syndrome. This disorder is caused by mechanical changes in the position of the child's abdominal contents that result from the lengthening of the child's body. It results in a syndrome of emesis and abdominal distention that is similar to that which occurs with intestinal obstruction or paralytic ileus. Postoperative vomiting among children with body casts or among those who have undergone spinal fusion warrants attention because of the possibility of superior mesenteric artery syndrome. Therefore, the remaining options are incorrect.

207. A licensed practical nurse (LPN) is bathing a neonate and notices small dark tufts of fine hair on the neonate's lower back. The LPN should take which best action? 1. Notify the registered nurse of the finding. 2. Assess for other associated anomalies and document carefully. 3. Tell the mother and father that this may indicate spina bifida. 4. Recognize that this is normal in the neonate and continue the bath

Notify the registered nurse of the finding. Rationale: The legal role of the LPN is to practice under the supervision of the registered nurse. In this instance, the tuft of hair may be indicative of a spinal anomaly, and the registered nurse should be notified of the finding. It is inappropriate to discuss abnormal findings with the parents because this is the responsibility of the health care provider, if an anomaly is suspected or diagnosed. The LPN should take the priority intervention of notifying the registered nurse before documenting in the chart.

268. A nurse has provided dietary instructions to the mother of a child with celiac disease. The nurse determines that further instructions are needed if the mother states that she will include which of the following in the child's nutritional plan? 1. Rice 2. Corn 3. Oatmeal 4. Vitamin supplements

Oatmeal Rationale: Dietary management is the mainstay of treatment for the child with celiac disease. All wheat, rye, barley, and oats should be eliminated from the diet and replaced with corn and rice. Vitamin supplements, especially fat-soluble vitamins and folate, may be needed in the early period of treatment to correct deficiencies.

281. A mother brings her 5-month-old daughter into the pediatrician's office with complaints that the child has been vomiting during feedings. The mother also states that the child is sometimes very fussy. The nurse's initial action would be to: 1. Assess the child's growth status. 2. Obtain a complete history of the child's feeding habits. 3. Assess whether any other children in the family have had the same problem. 4. Explain to the mother that the health care provider will prescribe a barium swallow and upper gastrointestinal (GI) series.

Obtain a complete history of the child's feeding habits. Rationale: In most situations, a complete history and physical examination of the child is the initial step in diagnosing gastroesophageal reflux disease. The child's feeding habits will give the nurse an indicator of the growth status. The child is weighed and measured after the initial interview is completed with the parent. Hereditary factors are not the priority. Further diagnostic studies may be ordered but only after a complete history is obtained.

226. A mother arrives at the emergency department with her child and a diagnosis of epiglottitis is documented. Which of the health care provider's prescriptions would be important for the nurse to question? 1. Obtain a throat culture. 2. Obtain axillary temperatures. 3. Administer humidified oxygen. 4. Administer antipyretics for fever.

Obtain a throat culture. Rationale: The throat of a child with suspected epiglottitis should not be examined or cultured, because any stimulation with a tongue depressor or culture swab could cause laryngospasm and complete airway obstruction. Humidified oxygen and antipyretics are components of the treatment. Axillary rather than oral temperatures should be taken.

71. Which of the following is the primary clinical manifestation of acute renal failure? Oliguria Hematuria Proteinuria Bacteriuria

Oliguria Oliguria is the primary clinical symptom of acute renal failure. Generally, urinary output is less than 1 ml/kg/hr. Hematuria, proteinuria, and bacteriuria may be present in renal disease, but they are not the primary manifestations of acute renal failure.

167. An infant returns to the nursing unit after the surgical repair of a cleft lip located on the right side of the lip. The best position in which to place this infant at this time is: 1. A flat position 2. A prone position 3. On his or her left side 4. On his or her right side

On his or her left side Rationale: After the repair of a cleft lip, the infant should be positioned on the side opposite to the repair to prevent contact of the suture lines with the bed linens. In this case, it is best to place the infant on his or her left side. Additionally, the flat or prone position can result in aspiration if the infant vomits.

76. Therapeutic management of the child with acute diarrhea and dehydration usually begins with which of the following? Clear liquids such as fruit juice and soft drinks Adsorbents, such as kaolin and pectin Oral rehydration solution Antidiarrheal medications such as paregoric

Oral rehydration solution Oral rehydration solution is the first treatment for acute diarrhea. Clear liquids are not recommended because they usually have a high carbohydrate content, low electrolyte content, and high osmolality. Adsorbents are not recommended. Antidiarrheals are not recommended because they do not get rid of pathogens.

296. A nursing instructor asks the nursing student to plan and conduct a clinical conference on phenylketonuria (PKU). The student researches the topic and plans to include which of the following in the conference? 1. PKU is an autosomal dominant disorder. 2. PKU results in central nervous system (CNS) damage. 3. Some state laws require routine screening of all newborn infants for PKU. 4. Treatment includes dietary restriction of sodium.

PKU results in central nervous system (CNS) damage. Rationale: PKU is an autosomal recessive disorder. Treatment includes dietary restriction of phenylalanine intake (not sodium). PKU is a genetic disorder that results in CNS damage from toxic levels of phenylalanine in the blood. All 50 states require routine screening of all newborn infants for PKU.

182. A child is admitted to the hospital with a probable diagnosis of nephrotic syndrome. Which findings would the nurse expect to observe? Select all that apply. 1. Pallor 2. Edema 3. Anorexia 4. Proteinuria 5. Weight loss 6. Decreased serum lipids

Pallor Edema Anorexia Proteinuria Rationale: Nephrotic syndrome is a kidney disorder that is characterized by massive proteinuria, hypoalbuminemia, edema, elevated serum lipids, anorexia, and pallor. The urine volume is decreased, and the urine is dark and frothy in appearance. The child with this condition gains weight.

194. A 4-year-old child is hospitalized with a suspected diagnosis of Wilms' tumor. The nurse assists with developing a plan of care. The nurse questions which intervention that is written in the plan of care? 1. Palpating the abdomen for a mass 2. Checking the urine for the presence of hematuria 3. Monitoring the blood pressure for the presence of hypertension 4. Monitoring the temperature for the presence of a kidney infection

Palpating the abdomen for a mass Rationale: Wilms' tumor is an intra-abdominal and kidney tumor. If Wilms' tumor is suspected, the mass should not be palpated. Excessive manipulation can cause seeding of the tumor and thus cause the spread of the cancerous cells. Hematuria, hypertension, and fever are clinical manifestations that are associated with Wilms' tumor.

154. A child has been diagnosed with Reye's syndrome. The nurse understands that a major symptom associated with Reye's syndrome is: 1. Persistent vomiting 2. Protein in the urine 3. Symptoms of hyperglycemia 4. A history of a Staphylococcus infection .

Persistent vomiting Rationale: Persistent vomiting is a major symptom that is associated with increased intracranial pressure (ICP). Options 2, 3, and 4 are incorrect. Protein is not present in the urine. Reye's syndrome is related to a history of viral infections, and hypoglycemia is a symptom of this disease

192. A nurse is assigned to care for a child who is in skeletal traction. The nurse avoids which of the following when caring for the child? 1. Keeping the weights hanging freely 2. Ensuring that the ropes are in the pulleys 3. Placing the bed linens on the traction ropes 4. Ensuring that the weights are out of the child's reach

Placing the bed linens on the traction ropes Rationale: Bed linens should not be placed on the traction ropes because of the risk of disrupting the traction apparatus. Options 1, 2, and 4 are appropriate measures when caring for a child who is in skeletal traction.

69. The clinical manifestations of nephrotic syndrome include which of the following? Hematuria, bacteriuria, and weight gain Gross hematuria, albuminuria, and fever Hypertension, weight loss, and proteinuria Proteinuria, hypoalbuminemia, and edema

Proteinuria, hypoalbuminemia, and edema Edema, proteinuria, hypoalbuminemia, and hypercholesterolemia are the clinical manifestations of nephrotic syndrome in children. Bacteriuria is not a diagnostic criterion for nephrotic syndrome. Fever is not associated with nephrotic syndrome. Weight gain occurs secondary to the edema.

246. A child with cerebral palsy (CP) is working to achieve maximum potential for locomotion, self-care, and socialization in school. The nurse would work with the child to meet these goals by: 1. Keeping the child in a special education classroom with other children with similar disabilities 2. Laying the child in the supine position with a 30-degree elevation of the head to facilitate feeding 3. Removing ankle-foot orthoses and braces once the child arrives at school 4. Placing the child on a wheeled scooter board

Placing the child on a wheeled scooter board Rationale: Option 4 provides the child with maximum potential in locomotion, self-care, and socialization. The child can move around independently on the abdomen anywhere the child wants to go and can interact with others as desired. Orthoses must be used all the time to aid locomotion (option 3). Option 1 does not provide for maximum socialization and normalization; rather, children with CP need to be mainstreamed as much as cognitively able. Not all children with CP are intellectually challenged. Option 2 does not provide for normalization in self-care. Just as children without CP sit up and use assistive devices when eating, so should children with CP.

84. A pediatric patient at risk for developing complications of immobility during the postoperative period should have which measures incorporated into the plan of care? (Select all that apply.) Place in supine position with extremities in alignment. Plan for play activities as tolerated. Include range of motion, both active and passive as tolerated. Limit nutritional intake until previous mobility pattern is restored. Maintain hydration during postoperative period.

Plan for play activities as tolerated. Include range of motion, both active and passive as tolerated. Maintain hydration during postoperative period. Although extremities should be maintained in proper alignment, the individual should not be placed in the supine position but rather placed in an upright posture when possible. Planning for play, performing range of motion exercise and maintaining hydration are all important elements of care to help prevent complications of immobility. Nutritional intake should include high protein, high fiber food sources.

223. A nurse is reviewing the laboratory results of a child scheduled for a tonsillectomy. Which laboratory value would be significant to review? 1. Creatinine 2. Urinalysis 3. Platelet count 4. Blood urea nitrogen (BUN)

Platelet count Rationale: Before the surgical procedure, the child is assessed for signs of active infection and for redness and exudate of the throat. Because the tonsillar area is so vascular, postoperative bleeding is a concern. The prothrombin (PT), partial thromboplastin time (PTT), platelet count, hemoglobin and hematocrit (H&H), white blood cell (WBC) count, and urinalysis are performed preoperatively. The platelet count result would identify a potential for bleeding. The BUN and creatinine would not determine the potential for bleeding but rather evaluate renal function.

56. The pediatric clinic nurse completes an assessment on a 4-month-old infant brought in because the parents are concerned that something is "just not right" with their baby. The nurse should alert the health care provider to which assessment findings? (Select all that apply.) Inability to sit up without support. Poor head control and clenched fists. Inability to crawl. Failure to smile. Extreme irritability.

Poor head control and clenched fists. Failure to smile. Extreme irritability. The infant would not be expected to sit up without support until 6 or 7 months old. Crawling would not be an expected finding in a 4-month-old infant. Early signs of cerebral palsy include: • Failure to meet any developmental milestones such as rolling over, raising head, sitting up, crawling • Persistent primitive reflexes such as Moro, asymmetrical tonic neck reflex • Poor head control (head lag) and clenched fists after 3 months old • Stiff or rigid arms or legs; scissoring legs • Pushing away or arching back; stiff posture • Floppy or limp body posture, especially while sleeping • Inability to sit up without support by 8 months • Using only one side of the body or only the arms to crawl • Feeding difficulties • Persistent gagging or choking when fed • After 6 months old, tongue pushing soft food out of the mouth • Extreme irritability or crying • Failure to smile by 3 months old • Lack of interest in surroundings

134. A nurse is caring for a 2-year-old child with tetralogy of Fallot (TOF) who is scheduled for surgery in 24 hours. What intervention is the most important for the nurse to include in the plan of care?

Position the child with knees to the chest Rationale: TOF consists of four congenital anomalies: pulmonic stenosis, intraventricular septal defect, overriding aorta, and right ventricular hypertrophy. Interventions for care include high flow oxygen, morphine, beta-blockers and positioning with knees to chest

229. The mother of a child arrives at the clinic because the child has been experiencing scratchy, red, and swollen eyes. The nurse notes a discharge from the eyes and a culture is sent to the laboratory for analysis. Chlamydial conjunctivitis is diagnosed. Based on this diagnosis, which of the following would require further investigation? 1. Possible trauma 2. Possible sexual abuse 3. The presence of an allergy 4. The presence of a respiratory infection

Possible sexual abuse Rationale: A diagnosis of chlamydial conjunctivitis in a non-sexually active child should signal the health care provider to assess the child for possible sexual abuse. Allergy, infection, and trauma can cause conjunctivitis but not chlamydial conjunctivitis.

241. A nurse is assigned to care for a 2-year-old child who has been admitted to the hospital for surgical correction of cryptorchidism. The highest priority in the postoperative plan of care for this child is to: 1. Force oral fluids. 2. Prevent tension on the suture. 3. Test the urine for glucose. 4. Encourage coughing.

Prevent tension on the suture. Rationale: When a child returns from surgery, the testicle is held in position by an internal suture that passes through the testes and scrotum and is attached to the thigh. It is important not to dislodge this suture. Depending on the type of anesthesia used, option 4 may be appropriate but is not the priority. Although adequate hydration is important to maintain, fluids should not be forced. Testing urine for glucose is not related to this type of surgery.

177. The child with cryptorchidism is being discharged after orchiopexy, which was performed on an outpatient basis. The nurse informs the parents about which priority care measure? 1. Measuring intake and output 2. Administering anticholinergics 3. Preventing infection at the surgical site 4. Applying cold, wet compresses to the surgical site

Preventing infection at the surgical site Rationale: The most common complications associated with orchiopexy are bleeding and infection. The parents are instructed in postoperative home care measures, including the prevention of infection, pain control, and activity restrictions. The measurement of intake and output is not required. Anticholinergics are prescribed for the relief of bladder spasms; they are not necessary after orchiopexy. Cold, wet compresses are not prescribed. The moisture from a wet compress presents a potential for infection.

93. Which of the following results when ice is applied immediately after a soft tissue injury, such as a sprained ankle? Increases the pain threshold Increases metabolism in the tissues Produces deep tissue vasodilation Leads to release of more histamine-like substances

Produces deep tissue vasodilation Nine to 15 minutes of ice exposure produces deep tissue vasodilation without increased metabolism. Ice has a rapid cooling effect on tissues that reduces pain. The decreased temperature slows metabolism, thus reducing tissue oxygen requirements. Fewer histamine-like substances are released.

242. A nurse is reviewing the health record of a child with a diagnosis of celiac disease. Which clinical manifestation should the nurse expect to note documented in the health record? 1. Frothy diarrhea 2. Profuse watery diarrhea and vomiting 3. Foul-smelling ribbon stools 4. Diffuse abdominal pain unrelated to meals or activity

Profuse watery diarrhea and vomiting Rationale: Celiac disease causes profuse watery diarrhea and vomiting. Option 1 is a clinical manifestation of lactose intolerance. Option 3 is a clinical manifestation of Hirschsprung's disease. Option 4 is a clinical manifestation of irritable bowel syndrome.

260. A nurse reviews the record of a 1-year-old child seen in the clinic and notes that the health care provider has documented a diagnosis of celiac crisis. Which of the following symptoms would the nurse expect to note in this condition? 1. Anorexia 2. Joint pain 3. Profuse, watery diarrhea 4. Constipation

Profuse, watery diarrhea Rationale: Clinical manifestations associated with celiac crisis include profuse, watery diarrhea and vomiting that quickly lead to severe dehydration and metabolic acidosis. The cause of the crisis is usually infection or hidden sources of gluten. The child may require intravenous fluids to correct fluid and acid-base imbalances, albumin to treat shock, and corticosteroids to decrease severe mucosal inflammation.

169. A nurse is reviewing the record of a child with a diagnosis of pyloric stenosis. Which data would the nurse expect to note as having been documented in the child's record? 1. Watery diarrhea 2. Projectile vomiting 3. Increased urine output 4. Vomiting large amounts of bile

Projectile vomiting Rationale: Clinical manifestations of pyloric stenosis include projectile, nonbilious vomiting; irritability; hunger and crying; constipation; and signs of dehydration, including a decrease in urine output.

41. Which measure is important in managing hypercalcemia in a child who is immobilized? Promoting adequate hydration Changing position frequently Encouraging a diet high in calcium Providing a diet high in protein and calories

Promoting adequate hydration Hydration is extremely important to help remove the excess calcium from the body. This can help prevent hypercalcemia. Changing position frequently will help manage skin integrity but will not affect calcium levels. The calcium will not be incorporated into bone because of the lack of weight bearing. The child is at risk of developing hypercalcemia. The child's metabolism is slower because of the immobilization. A diet with sufficient calories and nutrients for healing is important.

306. A nurse is assisting in developing a plan of care for a 10-year-old child diagnosed with acute glomerulonephritis. Following review of the plan of care, the nurse determines that which of the following is the priority for the child? 1. Restricting oral fluids 2. Allowing the child to play with the other children in the playroom 3. Promoting bedrest 4. Encouraging visits from friends

Promoting bedrest Rationale: Bedrest is required during the acute phase, and activity is gradually increased as the condition improves. Providing for quiet play according to the developmental stage of the child is important. Fluids should not be forced or restricted. Visitors should be limited to allow for adequate rest.

202. After a tonsillectomy, a child is brought to the pediatric unit. The nurse places the child in which appropriate position? 1. Prone 2. Supine 3. Trendelenburg's 4. High Fowler's

Prone Rationale: The child should be placed in a prone or side-lying position after tonsillectomy to facilitate drainage. Options 2, 3, and 4 will not achieve this goal.

258. A nurse is assigned to care for a child with hypertrophic pyloric stenosis who is scheduled for pyloromyotomy. Which of the following positions would the nurse place the child in during the preoperative period? 1. Prone with the head of the bed elevated 2. Prone with the head of the bed lowered to promote drainage 3. Supine with the head of the bed at a 30-degree angle 4. Supine with the head of the bed at a 45-degree angle

Prone with the head of the bed elevated Rationale: In the preoperative period, the infant is positioned prone with the head of the bed elevated to reduce the risk of aspiration. Options 2, 3, and 4 are inappropriate positions to prevent this risk.

274. A nurse is reviewing the record of a child admitted to the hospital with nephrotic syndrome. Which finding would the nurse expect to note documented in the record? 1. Proteinuria 2. Weight loss 3. Increased appetite 4. Hyperalbuminemia

Proteinuria Rationale: The term "nephrotic syndrome" refers to a kidney disorder characterized by proteinuria, hypoalbuminemia, and edema. The child experiences fatigue, anorexia, increased weight, abdominal pain, and a normal blood pressure.

137. A toddler is admitted to the facility with nephrotic syndrome. The nurse carefully monitors the toddler's fluid intake and output and checks urine specimens regularly with a reagent strip. Which finding is the nurse most likely to see?

Proteinuria Rationale: the glomerular membrane of the kidneys becomes permeable to proteins, resulting in massive proteinuria

128. In working with parents who have a child diagnosed with cerebral palsy, which therapeutic management goals should be included in the plan of care? (Select all that apply.) Limit socialization to similar type affected children. Provide educational opportunities that are individualized to children's needs and abilities. To help support and maintain location, communication and self-help skills. To correct body image perception. To integrate motor function.

Provide educational opportunities that are individualized to children's needs and abilities. To help support and maintain location, communication and self-help skills. To integrate motor function. The five broad goals of therapeutic management in reference to cerebral palsy include: establishing locomotion, communication and self-help skills, gaining optimal appearance and integration of motor function, correcting associated defects as early and effectively as possible, providing individualized educational opportunities for the child and promoting socialization experiences with both effected and non-effected children.

221. Choose the interventions that a nurse would include when writing a care plan for a child with hepatitis? Select all that apply. 1. Providing a low-fat, well-balanced diet 2. Notifying the health care provider if jaundice is present 3. Teaching the child effective hand washing techniques 4. Scheduling play time in the playroom with other children 5. Instructing the parents about the risks associated with taking medications 6. Arranging for indefinite home schooling because the child will not be able to return to school

Providing a low-fat, well-balanced diet Teaching the child effective hand washing techniques Instructing the parents about the risks associated with taking medications Rationale: Because hepatitis can be viral, standard precautions should be instituted in the hospital. The child should be discouraged from sharing toys, so playtime in the playroom with other children is not part of the plan of care. The child will be allowed to return to school 1 week after the onset of jaundice, so indefinite home schooling would not need to be arranged. Jaundice is an expected finding with hepatitis and would not warrant notification of the health care provider. Provision of a low-fat, well-balanced diet is recommended. Parents are cautioned about administering any medication to the child because normal doses of many medications may become dangerous because of the liver's inability to detoxify and excrete them. Handwashing is the single most effective measure in control of hepatitis in any setting, and effective handwashing can prevent the compromised child from picking up an opportunistic type of infection.

136. A preschool-age child with sickle cell anemia is admitted to the health care facility in vaso-occlusive crisis after developing a fever and joint pain. What is the nurse's highest priority when caring for this child?

Providing fluids Rationale: During a vaso-occlusive crisis, sickle-shaped red blood cells (RBCs) clump together and obstruct blood vessels, causing ischemia and tissue damage. Therefore, the highest priority is providing I.V. and oral fluids, which promotes hemodilution and aids the free flow of RBCs through blood vessels

24. A child is receiving cyclosporine following a kidney transplant. The nurse should include which information in the teaching plan about this medication? (Select all that apply.) 1. Optimal time to take medication to decrease pain 2. Recommended foods to take with medication to enhance boosting of immunity 3. Purpose of medication is to suppress rejection 4. How to palpate pulses to check for improved circulation 5. Frequent handwashing

Purpose of medication is to suppress rejection Frequent handwashing Cyclosporine is given to suppress rejection. Cyclosporine does not decrease pain, boost immunity, or improve circulation. When taking this medication, it is important to avoid others with contagious illnesses and to wash hands often, because it is an immunosuppressant medication.

123. Which parameters would confirm clinical diagnosis of urinary infections (UTI) in young children? (Select all that apply.) Fever Pyuria Clean catch specimen reported as being cloudy in appearance. 50,000 or greater colonies per mL indicating uropathic organism.

Pyuria Clean catch specimen reported as being cloudy in appearance. Confirmation of a UTI is based on results of properly collected specimen indicating pyuria and noted number of colony forming units of a specified uropathic organism. Cloudy appearance does not by itself provide confirmatory clinical evidence. Fever may or may not be associated with the presence of UTI.

86. The nurse stops to assist a child who has been hit by a car while riding a bicycle. Someone has activated the emergency medical system. Until paramedics arrive, the nurse should consider which of the following in caring for this child who has experienced severe trauma? Rapid assessment should begin with ABC status: airway, breathing, and circulation. Assessment should begin with the area injured; assessment of other areas can wait. The possibility of spinal cord injury should be ruled out before transporting the child to the hospital. Temperature maintenance is more difficult than in adults because young children have a larger surface area related to body mass.

Rapid assessment should begin with ABC status: airway, breathing, and circulation. The first priority is always airway, breathing, and circulation. Assessment of the injured area occurs after the child's cardiopulmonary status has been addressed. Transport can occur by immobilizing the cervical spine. The head is maintained in a neutral position, and movement of the head or body is not allowed in any direction. Infants have the greatest discrepancy in body surface areas. Children old enough to ride bikes have similar body proportions to adults.

44. Which is characteristic of fractures in children? Fractures rarely occur at the growth plate site, because it absorbs shock well. Rapidity of healing is inversely related to the age of the child. Pliable bones of growing children are less porous than those of the adult. Periosteum of a child's bone is thinner, weaker, and has less osteogenic potential compared with that of the adult.

Rapidity of healing is inversely related to the age of the child. The cartilage epiphyseal plate is the weakest point of the long bone. Therefore, it is a frequent site of damage. Fractures heal in children in less time than they do in adults. As the child ages, the healing time increases. The periosteum is thickened, and there is a great production of osteoclasts when a bone injury occurs. Bone healing in children is rapid because of the thickened periosteum and generous blood supply.

114. When considering Crohn's and Ulcerative Colitis (UC) as disease states, which clinical symptoms may appear to be common presentations in both? Rashes and joint pain Rectal bleeding Growth restriction Fistulas and strictures

Rashes and joint pain Rashes and joint pain are common presentations in both Crohn's and UC. Rectal bleeding is more common in UC. Growth restriction, fistulas and strictures are more common in Crohn's.

166. A nurse is monitoring for signs of dehydration in a 1-year-old child who has been hospitalized for diarrhea and prepares to take the child's temperature. Which method of temperature measurement should be avoided? 1. Rectal 2. Axillary 3. Electronic 4. Tympanic

Rectal Rationale: Rectal temperature measurements should be avoided if diarrhea is present. The use of a rectal thermometer can stimulate peristalsis and cause more diarrhea. Axillary or tympanic measurements of temperature would be acceptable. Most measurements are performed via electronic devices.

173. A nurse is assigned to care for a child who is suspected of having glomerulonephritis. The nurse reviews the child's record and notes that which finding is associated with the diagnosis of glomerulonephritis? 1. Hypotension 2. Red-brown urine 3. Low urinary specific gravity 4. A low blood urea nitrogen (BUN) level

Red-brown urine Rationale: Gross hematuria resulting in dark, smoky, cola-colored or red-brown urine is a classic symptom of glomerulonephritis, and hypertension is also common. A mid to high urinary specific gravity is associated with glomerulonephritis. BUN levels may be elevated.

43. What is the rationale for elevating an extremity after a soft tissue injury such as a sprained ankle? Increases the pain threshold. Increases metabolism in the tissues. Produces a deep tissue vasodilation. Reduces edema formation.

Reduces edema formation. This should have no effect on the pain threshold. This should not affect metabolism. Venous return to the heart, not vasodilation, is facilitated. Elevating the extremity uses gravity to facilitate venous return to reduce edema.

103. Which of the following is an important nursing consideration in the care of a child with celiac disease? Refer to a nutritionist for detailed dietary instructions and education. Help child and family understand that diet restrictions are usually only temporary. Teach proper hand washing and Standard Precautions to prevent disease transmission. Suggest ways to cope more effectively with stress to minimize symptoms.

Refer to a nutritionist for detailed dietary instructions and education. 1. The main consideration is helping the child adhere to dietary management. Considerable time is spent in explaining to the child and parents the disease process, the specific role of gluten in aggravating the condition, and those foods that must be restricted. Referral to a nutritionist would help in this process. 2. The most severe symptoms usually occur in early childhood and adult life. Dietary avoidance of gluten should be lifelong. 3. Celiac disease is not transmissible. 4. Celiac disease is not stress related.

99. Therapeutic management of most children with Hirschsprung disease is primarily which of the following? Daily enemas Low-fiber diet Permanent colostomy Removal of affected piece of bowel

Removal of affected piece of bowel Most children with Hirschsprung disease require surgical rather than medical management. Surgery is done to remove the aganglionic portion of the bowel, relieve obstruction, and restore normal bowel motility and function of the internal anal sphincter. 1. Preoperative management may include enemas and low-fiber, high-calorie, high-protein diet, until the child is physically ready for surgery. 2. Preoperative management may include enemas and low-fiber, high-calorie, high-protein diet, until the child is physically ready for surgery. 3. The colostomy that is created in Hirschsprung disease is usually temporary.

218. A male child who had surgery to correct hypospadias is seen in a health care provider's office for a well-baby check-up. The nurse provides instructions to the mother, knowing that which long-term complication is associated with hypospadias? 1. Infertility 2. Renal anomalies 3. Erectile dysfunction 4. Decreased urinary output

Renal anomalies Rationale: The nurse should ask the child's parents about the child's kidney function because hypospadias may be associated with renal anomalies. The incorrect options are not associated with a long-term effect of hypospadias.

234. A nurse collecting data on a child suspects physical abuse. The nurse understands that which of the following is a primary and legal nursing responsibility? 1. Document the child's physical assessment findings accurately and thoroughly. 2. Report the case in which the abuse is suspected. 3. Refer the family to the appropriate support groups. 4. Assist the family in identifying resources and support systems.

Report the case in which the abuse is suspected. Rationale: The primary legal nursing responsibility when child abuse is suspected is to report the case. All 50 states require health care professionals to report all cases of suspected abuse. Although documentation of findings, assisting the family, and referring the family to appropriate resources and support groups are important, the primary legal responsibility is to report the case.

175. A nurse is planning care for a child with hemolytic-uremic syndrome (HUS). The child has been anuric and will be receiving peritoneal dialysis treatment. The nurse plans to: 1. Restrict fluids, as prescribed. 2. Administer analgesics, as prescribed. 3. Care for the arteriovenous (AV) fistula. 4. Encourage the intake of foods that are high in potassium.

Restrict fluids, as prescribed. Rationale: HUS is thought to be associated with bacterial toxins, chemicals, and viruses that cause acute renal failure in children. Clinical features of the disease include acquired hemolytic anemia, thrombocytopenia, renal injury, and central nervous system symptoms. A child with HUS who is undergoing peritoneal dialysis for the treatment of anuria will be on fluid restrictions. Pain is not associated with HUS, and potassium would be restricted rather than encouraged if the child was anuric. Peritoneal dialysis does not require an AV fistula (only hemodialysis does).

171. A nurse reinforces home-care instructions to the parents of a child with celiac disease. Which of the following food items would the nurse advise the parents to include in the child's diet? 1. Rice 2. Oatmeal 3. Rye toast 4. Wheat bread

Rice Rationale: Dietary management is the mainstay of treatment for celiac disease. All wheat, rye, barley, and oats should be eliminated from the diet and replaced with corn and rice. Vitamin supplements, especially fat-soluble vitamins and folate, may be required during the early period of treatment to correct deficiencies. These restrictions are likely to be lifelong, although small amounts of grains may be tolerated after the gastrointestinal ulcerations have healed.

261. A nurse is assisting a health care provider with an assessment of a child with a diagnosis of suspected appendicitis. In assessing the intensity and progression of the pain, the health care provider palpates the child at McBurney's point. The nurse understands that McBurney's point is located midway between the: 1. Right anterior inferior iliac crest and the umbilicus 2. Left anterior superior iliac crest and the umbilicus 3. Right anterior superior iliac crest and the umbilicus 4. Left anterior superior iliac crest and the umbilicus

Right anterior superior iliac crest and the umbilicus Rationale: McBurney's point is midway between the right anterior superior iliac crest and the umbilicus. It is usually the location of greatest pain in the child with appendicitis. Options 1, 2, and 4 are incorrect.

74. Which of the following is the viral pathogen that frequently causes acute diarrhea in young children? Giardia organisms Shigella organisms Rotavirus Salmonella organisms

Rotavirus Rotavirus is the viral pathogen that most frequently causes diarrhea in young children. Giardia is a bacterial pathogen that causes diarrhea. Shigella is a bacterial pathogen that is uncommon in the United States. Salmonella is a bacterial pathogen that causes diarrhea.

141. The nurse is assessing an adolescent who has been diagnosed with aplastic anemia. Which of the following should be the priority assessment for the nurse?

S/S of infection Rationale: lowering of all blood cells->causes neutropenia->which puts patient at risk for infection

285. An adolescent is admitted to the hospital with complaints of lower right abdominal pain. The health care provider prescribes laboratory tests to rule out ectopic pregnancy rather than appendicitis. Which of the following is most significant in ruling out an ectopic pregnancy? 1. Urinalysis 2. White blood count 3. C-reactive protein 4. Serum human chorionic gonadotropin

Serum human chorionic gonadotropin Rationale: The test to rule out an ectopic pregnancy is the serum human chorionic gonadotropin. The other tests may be prescribed to rule out appendicitis, but because the client is an adolescent it would be necessary to rule out an ectopic pregnancy as well. Urinalysis will rule out a urinary tract infection, and the white blood count and the C-reactive protein will rule out some other types of infection.

115. A child is exhibiting signs of clinical dehydration. Which laboratory value would support a diagnosis of hypertonic dehydration? Serum sodium level of 135 mEq/dL Plasma osmolality of 275 mOsm/L Calculation of loss of body fluid weight at 25 mL/kg Serum sodium level of 150 mEq/dL

Serum sodium level of 150 mEq/dL Hypertonic dehydration would result in an increase in serum sodium levels in proportion to fluid loss. Normal serum sodium level ranges between 135 and 145 mEq/dL. Normal plasma osmolality is within the 275 to 295 mOsm/L. Calculation of loss of body fluid weight in terms of moderate loss would be at 50 mL/kg with 100 mL/kg being severe.

100. A proton pump inhibitor (PPI) is ordered for an infant with gastroesophageal reflux. The nurse should include in the drug teaching that: the drug should be given 30 minutes before bedtime. three times a day dosing has maximum effect. the drug can be stopped once symptoms have resolved. several days may pass before full effect is reached.

Several days may pass before full effect is reached PPIs require several days to achieve the maximum effect. 1. Optimum administration time is 30 minutes before breakfast. This allows for peak plasma levels at mealtime. 2. Once daily dosing is usually recommended. 3. Continued administration is necessary to maintain effect.

66. Which of the following factors predisposes the urinary tract to infection? Increased fluid intake Short urethra in girls Ingestion of highly acidic juices Frequent emptying of the bladder

Short urethra in girls The short urethra, which measures approximately 2 cm (0.8 in) in girls and 4 cm (1.6 in) in mature women, provides a ready pathway for invasion of organisms. Increased fluid intake results in frequent emptying of the bladder, preventing urinary stasis. Urine is bacteriostatic at pH of 5.0. This is not achievable by ingestion of juice. Frequent emptying of the bladder helps prevent urinary tract infections.

293. A nurse is assisting in developing a plan of care for a child who will be returning from the operating room following a tonsillectomy. The nurse plans to place the child in which position on return from the operating room? 1. Side-lying 2. Trendelenburg's and on the right side 3. Supine 4. High Fowler's and on the left side

Side-lying Rationale: The child should be placed in a prone or side-lying position following tonsillectomy to facilitate drainage. Options 2, 3, and 4 will not facilitate drainage.

294. A nurse is assisting in admitting to the hospital a 4-month-old infant with a diagnosis of vomiting and dehydration. The nurse assists in developing a plan of care for the infant and suggests including in the plan to position the infant in a(n): 1. Prone position 2. Side-lying position 3. Modified Trendelenburg's position 4. Infant car seat with the head of the seat in a flat position

Side-lying position Rationale: The vomiting infant or child should be placed in an upright or side-lying position to prevent aspiration. The positions identified in options 1, 3, and 4 will increase the risk of aspiration if vomiting occurs.

219. A nurse assists in preparing a plan of care for the infant with bladder exstrophy. The nurse identifies which of the following immediate problems as the priority for the infant? 1. Infection 2. Elimination 3. Skin disruption 4. Lack of parental understanding

Skin disruption Rationale: In bladder exstrophy, the bladder is exposed and external to the body. The highest priority is skin disruption related to the exposed bladder mucosa. Although the infant needs to be monitored for elimination patterns and kidney function, this is not the priority concern for this condition. Lack of parenteral understanding related to the diagnosis and treatment of the condition will need to be addressed, but again is not the priority. Although infection related to the anatomically located defect can be a problem, it is not the immediate one.

40. The nurse manager on the orthopedic unit is preparing an in-service about types of traction at the next staff meeting. The nurse manager should include which information in the presentation? (Select all that apply.) Skeletal traction is most likely used when closed reduction is performed. Skin traction can be applied using a pulling mechanism attached with adhesive material. Soft, foam-backed traction straps are used to distribute manual traction pull. Pins are commonly used with skeletal traction. Manual traction involves using wires or tongs inserted through the diameter of the bone distal to the fracture.

Skin traction can be applied using a pulling mechanism attached with adhesive material. Pins are commonly used with skeletal traction. Types of traction include: Manual traction—Applied to the body part by the hands placed distal to the fracture site. Manual traction may be provided during application of a cast but more commonly when a closed reduction is performed. Skin traction—Applied directly to the skin surface and indirectly to the skeletal structures. The pulling mechanism is attached to the skin with adhesive material or an elastic bandage. Both types are applied over soft, foam-backed traction straps to distribute the traction pull. Skeletal traction—Applied directly to the skeletal structure by a pin, wire, or tongs inserted into or through the diameter of the bone distal to the fracture.

63. What most accurately describes bowel function in children born with a myelomeningocele? Incontinence cannot be prevented. Enemas and laxatives are contraindicated. Some degree of fecal continence can usually be achieved. Colostomy is usually required by the time the child reaches adolescence.

Some degree of fecal continence can usually be achieved. Although a lengthy process, continence can be achieved with modification of diet, use of laxatives, and/or enemas. These are part of the strategy to achieve continence. There is no general contraindication. With diet modification and regular toilet habits to prevent constipation and impaction, some degree of fecal continence can be achieved. Colostomy usually is not required.

145. When obtaining a health history from the mother of a 7-year-old child diagnosed with acute rheumatic fever, the nurse should focus questions to determine if the child was recently ill with which condition?

Sore throat Rationale: generally follows infection with streptococci within about 2 weeks

254. Following a cleft lip repair, the nurse provides instructions to the parents regarding cleaning of the lip repair site. Which of the following solutions would the nurse use in demonstrating this procedure to the parents? 1. Tap water 2. Sterile water 3. Full-strength hydrogen peroxide 4. Half-strength hydrogen peroxide

Sterile water Rationale: The lip repair site is cleansed with sterile water using a cotton swab after feeding and as prescribed. The parents should be instructed to use a rolling motion from the suture line out. The parents should also demonstrate performance of the correct procedure to the nurse.

276. The nurse is caring for a pediatric client in skin traction. To prevent skin breakdown, the best nursing intervention for this child is to: 1. Vigorously massage bony prominences every 4 hours. 2. Replace the elastic bandage on skin traction every 8 hours. 3. Stimulate circulation with gentle massage over pressure areas. 4. Change the child's position at least every 4 hours to relieve pressure.

Stimulate circulation with gentle massage over pressure areas. Rationale: Nonadhesive straps and/or elastic bandage on skin traction are replaced when permitted and/or when absolutely necessary. Circulation should be stimulated with gentle, not vigorous, massage over pressure areas. The child's position should be changed at least every 2 hours to relieve pressure.

276. The nurse is caring for a pediatric client in skin traction. To prevent skin breakdown, the best nursing intervention for this child is to: 1. Vigorously massage bony prominences every 4 hours. 2. Replace the elastic bandage on skin traction every 8 hours. 3. Stimulate circulation with gentle massage over pressure areas. 4. Change the child's position at least every 4 hours to relieve pressure.

Stimulate circulation with gentle massage over pressure areas. Rationale: Nonadhesive straps and/or elastic bandage on skin traction are replaced when permitted and/or when absolutely necessary. Circulation should be stimulated with gentle, not vigorous, massage over pressure areas. The child's position should be changed at least every 2 hours to relieve pressure.

87. An important nursing consideration when caring for a child with juvenile idiopathic arthritis is which of the following? Apply ice packs to relieve stiffness and pain. Administer acetaminophen to reduce inflammation. Teach the child and family the correct administration of medications. Encourage range of motion exercises during periods of inflammation.

Teach the child and family the correct administration of medications. The management of juvenile idiopathic arthritis is primarily pharmacologic. The family should be instructed regarding administration of medications and the value of a regular schedule of administration to maintain a satisfactory blood level in the body. They need to know that nonsteroidal antiinflammatory drugs should not be given on an empty stomach and to be alert for signs of toxicity. Warm, moist heat is best for relieving stiffness and pain. Acetaminophen does not have antiinflammatory effects. Range of motion exercises should not be done during periods of inflammation.

94. A 4-year-old child is newly diagnosed with Legg-Calvé-Perthes disease. Nursing considerations include which of the following? Encourage normal activity for as long as possible. Explain the cause of the disease to the child and family. Prepare the child and family for long-term, permanent disabilities. Teach the family the care and management of the corrective appliance.

Teach the family the care and management of the corrective appliance. The family needs to learn the purpose, function, application, and care of the corrective device and the importance of compliance to achieve the desired outcome. The initial therapy is rest and non-weight-bearing activity, which help reduce inflammation and restore motion. Legg-Calvé-Perthes is a disease of unknown etiology. A disturbance of circulation to the femoral capital epiphysis produces an ischemic aseptic necrosis of the femoral head. The disease is self-limiting, but the ultimate outcome depends on early and efficient therapy and the age of the child at onset.

2. The nurse is discussing home care with a mother whose 6-year-old child has hepatitis A. What action should the nurse include? Informing her that bed rest is important until 1 week after the icteric phase Telling her that the child should not return to school until 3 weeks after icteric phase Giving reassurance that hepatitis A cannot be transmitted to other family members Teaching infection control measures to family members

Teaching infection control measures to family members

206. A licensed practical nurse is providing care for a child with hydrocephalus who has had a ventriculoperitoneal shunt revision. Which data collection finding should be reported to the registered nurse immediately? 1. Temperature 100.9° F 2. Pulse 78 beats per minute 3. Blood pressure 110/70 mm Hg 4. Respirations 22 breaths per minute

Temperature 100.9° F Rationale: Fever may be an indication of an infection of the shunt, which is the primary concern in the postoperative period, related to a shunt insertion. All of the other vital signs are normal findings for this child.

174. A nurse is reviewing the health record of a child who has been recently diagnosed with glomerulonephritis. Which finding noted in the child's record is associated with the diagnosis of glomerulonephritis? 1. The child fell off a bike and onto the handlebars. 2. The child has had nausea and vomiting for the last 24 hours. 3. The child had urticaria and itching for 1 week before diagnosis. 4. The child had a streptococcal throat infection 2 weeks before diagnosis.

The child had a streptococcal throat infection 2 weeks before diagnosis. Rationale: Group A β-hemolytic streptococcal infection is a cause of glomerulonephritis. The child often becomes ill with streptococcal infection of the upper respiratory tract and then develops symptoms of acute poststreptococcal glomerulonephritis after an interval of 1 to 2 weeks. The data presented in options 1, 2, and 3 are unrelated to a diagnosis of glomerulonephritis.

129. A woman who is 6 weeks pregnant tells the nurse that she is worried her baby might have spina bifida because of a family history. What should the nurse's response be based on? There is no genetic basis for the defect. Prenatal detection is not possible yet. Chromosomal studies done on amniotic fluid can diagnose the defect prenatally. The concentration of α-fetoprotein in amniotic fluid can potentially indicate the presence of the defect prenatally.

The concentration of α-fetoprotein in amniotic fluid can potentially indicate the presence of the defect prenatally. Fetal ultrasound and elevated concentrations of α-fetoprotein in amniotic fluid many indicate the presence of anencephaly, myelomeningocele, or other neural tube defects. The origin of neural tube defects is unknown but appears to have a multifactorial inheritance pattern. Prenatal detection is possible through amniotic fluid or chorionic villi sampling. There are no chromosomal studies currently that can diagnose spina bifida prenatally.

83. Basic treatment of musculoskeletal soft tissue injuries involves the use of RICE modalities during the first 12 to 24 hours. Which option would be included in RICE? Reusing chemical ice packs through the treatment period. Applying ice for 1 hour intervals throughout the treatment period. Pillows can used to achieve elevation of the extremity below the level of the heart. The extremity should be maintained in proper alignment and activity should be limited.

The extremity should be maintained in proper alignment and activity should be limited. R refers to rest and the involved extremity should be maintained in proper alignment with activity (range of motion) limited in order to prevent further damage. Reusing chemical ice packs is not acceptable. Ice should be applied for no more than 30 minutes at a time. Pillows can be used to elevate the extremity but it should be done above the level of the heart. To facilitate venous return and prevent fluid accumulation.

211. A nurse provides instructions to the parents of an infant with hip dysplasia regarding care of the Pavlik harness. Which instruction provided by the nurse is accurate? 1. The harness must be worn 8 hours a day. 2. The infant should never be moved when out of the harness. 3. The harness needs to be removed to check the skin and for bathing. 4. The harness must be removed for diaper changes and for feeding.

The harness needs to be removed to check the skin and for bathing. Rationale: The harness should be worn 23 hours a day and should be removed only to check the skin and for bathing. The hips and buttocks should be supported carefully when the infant is out of the harness. The harness does not need to be removed for diaper changes or feedings.

244. A 2-year-old child is diagnosed with constipation. Which of the following describes a characteristic of this disorder? 1. Incomplete development of the anus 2. Invagination of a section of the intestine into the distal bowel 3. The infrequent and difficult passage of dry stools 4. The presence of fecal incontinence

The infrequent and difficult passage of dry stools Rationale: Constipation can affect any child at any time, although its incidence peaks at ages 2 to 3 years. Option 4 describes encopresis, which can develop as a result of constipation and is one of the major concerns regarding constipation. Encopresis generally affects preschool and school-age children. Option 1 describes imperforate anus, which is diagnosed in the neonatal period. Option 2 describes intussusception, which is the most common cause of bowel obstruction in children ages 3 months to 6 years.

263. A nurse is monitoring a newborn with a suspected diagnosis of imperforate anus. The nurse understands that which of the following is unassociated with this disorder? 1. The presence of stool in the urine 2. Failure to pass a rectal thermometer 3. Failure to pass meconium in the first 24 hours after birth 4. The passage of currant jelly-like stools

The passage of currant jelly-like stools Rationale: During the newborn assessment, imperforate anus should be easily identified visually. However, a rectal thermometer or tube may be necessary to determine patency if meconium is not passed in the first 24 hours after birth. The presence of stool in the urine or vagina should be reported immediately as an indication of abnormal anorectal development. Currant jelly-like stool is not a clinical manifestation of this disorder.

132. A 5-year-old child returns to the pediatric unit following a cardiac catheterization using the right femoral vein. The child has a thick elastoplast dressing. Which assessment finding requires immediate intervention?

The pedal pulse of the right leg isn't detectable. Rationale: Using the femoral vein during catheterization can cause the affected blood vessels to spasm or cause a blood clot to develop, altering circulation in the leg. The inability to detect the pedal pulse in the affected leg is an ominous sign and requires immediate intervention.

210. A nurse is providing information to the family of a child about a synthetic cast that has been applied to the child for the treatment of a clubfoot. Which information should the nurse provide to the mother? 1. The synthetic cast takes 24 hours to dry. 2. The synthetic cast is heavier than a plaster cast. 3. The synthetic cast is stronger than a plaster cast. 4. The synthetic cast allows for greater mobility than a plaster cast.

The synthetic cast allows for greater mobility than a plaster cast. Rationale: Synthetic casts dry quickly (in less than 30 minutes) and are lighter than plaster casts. Synthetic casts allow for greater mobility than a plaster cast. However, synthetic casts are not as strong as plaster casts and are more expensive.

256. A nurse provides feeding instructions to a mother of an infant diagnosed with gastroesophageal reflux (GER). To assist in reducing the episodes of emesis, the nurse tells the mother to: 1. Thin the feedings by adding water to the formula. 2. Thicken the feedings by adding rice cereal to the formula. 3. Provide less frequent, larger feedings. 4. Burp less frequently during feedings.

Thicken the feedings by adding rice cereal to the formula. Rationale: Small, more frequent feedings with frequent burping are often tried as the first line of treatment in GER. Feedings thickened with rice cereal may reduce episodes of emesis. Thickened feedings do not affect reflux time, however. If thickened formula is prescribed, 1 to 3 teaspoons of rice cereal per ounce of formula is most commonly used and may require cross-cutting the nipple. Options 1, 3, and 4 are incorrect.

282. The nurse in the newborn nursery is preparing to feed a newborn the first feeding of sterile water. During the feeding, the newborn suddenly begins to cough, choke, and become cyanotic. Based on these symptoms, the nurse might suspect that the newborn has which of the following conditions? 1. Atrial septal defect 2. Tracheoesophageal fistula 3. Bronchopulmonary dysplasia 4. Respiratory distress syndrome

Tracheoesophageal fistula Rationale: The first feeding a newborn receives is sterile water to assess whether the newborn might have one of the tracheoesophageal (TE) conditions. Although sterile water is more easily absorbed and causes less aspiration than formula, the newborn with a suspected TE fistula condition will cough and choke during feedings. These symptoms are not associated with the conditions noted in options 1, 3, or 4.

156. After a tonsillectomy, the child begins to vomit bright red blood. The initial nursing action would be to: 1. Turn the child to the side. 2. Notify the RN or health care provider (HCP). 3. Administer the prescribed antiemetic. 4. Maintain nothing-by-mouth (NPO) status.

Turn the child to the side. Rationale: After a tonsillectomy, if bleeding occurs, the child is turned to the side, and the RN or HCP is notified. An NPO status would be maintained, and an antiemetic may be prescribed; however, the initial nursing action would be to turn the child to the side.

126. Which presentation is found in a pediatric patient who has Brown-Sequard syndrome? Complete transection of the spinal cord Bilateral cord dysfunction with complete loss of sensation Unilateral cord lesion with alternate side deficits Transient loss of function

Unilateral cord lesion with alternate side deficits Brown-Sequard syndrome presents with unilateral cord lesion alternate side deficits. It does not involve a complete transection of the spinal cord. Motor and sensory deficits are not transient.

280. A nurse is collecting data from a child with a diagnosis of diabetes insipidus. Which clinical finding is consistent with this diagnosis? 1. Urinary output is increased. 2. Urinary output is decreased. 3. Serum sodium is decreased. 4. Urine specific gravity is increased.

Urinary output is increased. Rationale: A child with a diagnosis of diabetes insipidus experiences increased urinary output, increased serum sodium, and decreased urine specific gravity. Decreased urinary output, decreased serum sodium, and increased urine specific gravity are consistent with a diagnosis of syndrome of inappropriate antidiuretic hormone (SIADH).

228. A health care provider prescribes intravenous potassium for a child with hypertonic dehydration. The nurse assigned to assist in caring for the child would check which highest-priority item before administration of the potassium? 1. Weight 2. Urine output 3. Temperature 4. Blood pressure

Urine output Rationale: The priority assessment would be to check the status of urine output. Potassium should never be administered in the presence of oliguria or anuria. If urine output is less than 1 to 2 mL/kg/hr, it should not be administered. Although options 1, 3, and 4 may be a component of the data collected, they are not specifically related to the administration of this medication.

212. A nurse is assigned to care for a child with a spica cast. Which action should be avoided when caring for the child? 1. Observing for nonverbal signs of pain 2. Using pillows to elevate the head and shoulders 3. Checking neurovascular status of the extremities 4. Placing the child on a stretcher and bringing the child to the playroom

Using pillows to elevate the head and shoulders Rationale: Pillows should not be used to elevate the head or shoulders of a child in a body cast because the pillows will thrust the child's chest against the cast and cause discomfort and respiratory difficulty. Neurovascular checks are a critical component of care to ensure that the cast is not causing circulatory compromise. The nurse should observe for nonverbal signs of pain and ask the older child if pain is experienced. A ride on a stretcher to the playroom or around the hospital provides changes of position and scenery.

88. Immobilization causes which of the following effects on the cardiovascular system? Venous stasis Increased vasopressor mechanism Normal distribution of blood volume Increased efficiency of orthostatic neurovascular reflexes

Venous stasis The physiologic effects of immobilization, as a result of decreased muscle contraction, include venous stasis. This can lead to pulmonary emboli or thrombi. A decreased vasopressor mechanism results in orthostatic hypotension, syncope, decreased cerebral blood flow, and tachycardia. An altered distribution of blood volume is found with decreased cardiac workload and exercise tolerance. Immobilization causes a decreased efficiency of orthostatic neurovascular reflexes with an inability to adapt readily to the upright position and with pooling of blood in the extremities in the upright position.

23 A nurse is caring for an infant with a suspected urinary tract infection (UTI). Based on the nurse's knowledge of UTIs, which clinical manifestation would be observed? (Select all that apply.) 1. Vomiting 2. Jaundice 3. Swelling of the face 4. Persistent diaper rash 5. Failure to gain weight

Vomiting Persistent diaper rash Failure to gain weight Vomiting is a clinical manifestation observed in an infant with a urinary tract infection (UTI) and can be related to poor feeding. Persistent diaper rash is a clinical manifestation of UTI in an infant. Failure to gain weight is a clinical manifestation of UTI in an infant related to poor feeding and vomiting. Jaundice is not a clinical manifestation of UTI in an infant. Swelling of the face is not a clinical manifestation of UTI in an infant.

270. A nurse is caring for a 1-year-old child following a cleft palate repair. Which solution should the nurse use after feedings to cleanse the child's mouth? 1. Water 2. Diluted hydrogen peroxide 3. A soft lemon glycerin swab 4. Half-strength povidone-iodine (Betadine) solution

Water Rationale: Following a cleft palate repair, the mouth is rinsed with water after feedings to clean the palate repair site. Rinsing food and residual sugars from the suture line reduces the risk of infection. Options 2, 3, and 4 are incorrect procedures, and the solutions identified in these options should not be used.

48. An infant is born with one lower limb deficiency. When is the optimum time for the child to be fitted with a prosthetic device? As soon as possible after birth When the infant begins sitting up and can maintain balance At about age 12 to 15 months, when most children are walking At about 4 years, when the healthy limb is not growing so rapidly

When the infant begins sitting up and can maintain balance The device will not be useful until the child is developmentally ready to use the leg. This is the most optimum time for the child to be fitted with a prosthetic device. The child is ready to stand, and the prosthetic device will be integrated into his or her capabilities.This may be too late. The device should be provided when the child is showing readiness to stand. This is too late.

303. A nurse is assisting in planning discharge instructions to the mother of a child following orchiopexy, which was performed on an outpatient basis. Which of the following is the priority in the plan of care? 1. Pain control measures 2. Measurement of intake 3. Wound care 4. Cold and heat applications

Wound care Rationale: The most common complications associated with orchiopexy are bleeding and infection. Discharge instruction should include demonstration of proper wound cleansing and dressing and teaching parents to identify signs of infection such as redness, warmth, swelling, or discharge. Testicles will be held in a position to prevent movement, and great care should be taken to prevent contamination of the suture line. Analgesics may be prescribed but are not the priority, considering the options presented. Option 2 is not necessary. Option 4 is not a prescribed treatment measure.

125. Which statement is true regarding the genetic transmission of Duchenne muscular dystrophy (DMD)? Multiple gene expression X-linked recessive Autosomal dominant No carrier states exist

X-linked recessive DMD is transmitted as an X-linked recessive single gene trait. Carrier states do exist.

59. An 8-year-old female child is diagnosed with moderate cerebral palsy (CP). She recently began participation in a regular classroom for part of the day. Her mother asks the school nurse about having her daughter join the after-school Girl Scout troop. The nurse's response should be based on knowledge that: most activities such as Girl Scouts cannot be adapted for children with CP. after-school activities usually result in extreme fatigue for children with CP. trying to participate in activities such as Girl Scouts leads to lowered self-esteem in children with CP. after-school activities often provide children with CP opportunities for socialization and recreation.

after-school activities often provide children with CP opportunities for socialization and recreation. Most activities can be adapted for children. The child, family, and activity director should assess the degree of activity to ensure that it meets with the child's capabilities. A supportive environment will add to the child's self-esteem. Recreational outlets and after-school activities should be considered for the child who is unable to participate in athletic programs.

119. Urine specimen results for a pediatric patient note greater than 100,000 colony forming units (CFUs) but the patient denies any complaints with urination. Based on this information the nurse would suspect that the patient has subacute pyelonephritis. pyuria. asymptomatic bacteriuria. febrile UTI.

asymptomatic bacteriuria. Presence of a significant amount of CFUs without clinical symptoms indicate that the patient has asymptomatic bacteriuria. Subacute pyelonephritis would refer to a chronic infection of both upper and lower urinary tract. Pyuria refers to the presence of white blood cells in the urine. Febrile UTI refers to bacteriuria in the presence of fever with other associated clinical signs.

147. Which meal would be most appropriate for a 15-year-old with glomerulonephritis with severe hypertension?

baked chicken, rice, beans, orange juice Rationale: the best choice is no salty foods in the answer

117. A 4-year-old child has ingested a toxic dose of iron. The parent reports that the child vomited and complained of gastric pain an hour ago but "feels fine" now. The parent is not certain when the child ingested the iron tablets. The most appropriate recommendation by the nurse to the parent is to observe the child closely for 2 more hours. bring the child to the hospital immediately. administer activated charcoal. administer ipecac to induce vomiting if the child does not vomit again within 1 hour.

bring the child to the hospital immediately. The child should be transported to the hospital immediately for assessment and possible gastric lavage. The period of concern for complications of iron toxicity is from 30 minutes to 6 hours. Activated charcoal does not bind iron and, therefore, is not a course of treatment for this child. Ipecac is not recommended for poisonings.

85. A 3-year-old has just returned from surgery in a hip spica cast. The priority nursing intervention is to elevate the head of the bed. offer sips of water. check circulation, sensation, and motion of toes. turn the child to the right side, then the left side every 4 hours.

check circulation, sensation, and motion of toes. The chief concern is that the extremity may continue to swell. The circulation, sensation, and motion of the toes must be assessed to ensure that the cast does not become a tourniquet and cause complications. Elevating the head of the bed might help with comfort, but it is not a priority. The nurse must be observant to the risk of increased swelling in the extremities. Offering sips of water is acceptable once assessment of the extremities has been completed. The child's position should be changed every 2 hours. Positioning a child with a spica cast is important to prevent injury.

46. A youngster has just returned from surgery in a hip spica cast. The PRIORITY nursing intervention is to: elevate the head of the bed. check circulation. turn the child to the right side. offer sips of water.

check circulation. Elevating the head of the bed might help with comfort. The nurse must be observant to the risk of increased swelling in the extremities. The chief concern is that the extremity may continue to swell. This must be assessed to ensure that the cast does not become a tourniquet. The child's position should be changed every 2 hours. This is acceptable, but only after the assessment of the extremities is completed.

133. A child, age 4, is admitted with a tentative diagnosis of congenital heart disease. When assessment reveals a bounding radial pulse coupled with a weak femoral pulse, the nurse suspects that the child has:

coarctation of the aorta. Rationale: because it causes signs of peripheral hypoperfusion, such as a weak femoral pulse and a bounding radial pulse

38. Therapeutic management of the patient with systemic lupus erythematosus includes: cold salts to suppress the inflammatory process. a high-protein, low-salt diet. an exercise regimen to build up muscle strength and endurance. corticosteroids to control inflammation.

corticosteroids to control inflammation. This will not affect the inflammatory process. A balanced diet without exceeding caloric expenditures is recommended. Exercise should be done in moderation. Currently this is the primary mode of therapy.

143. The nurse reviews with the parents how to care for their child with sickle cell anemia at home. The nurse determines that the parents understand the basic principles of care when they state that they will:

encourage their child to drink as much liquid as possible. Rationale: Hydration and pain management is the best way to prevent sickle cell crisis

72. Dialysis or transplantation becomes necessary for chronic renal failure when anemia develops. acidosis develops. glomerular filtration rate falls below 50% of normal. glomerular filtration rate falls below 10% to 15% of normal.

glomerular filtration rate falls below 10% to 15% of normal. Treatment with dialysis or transplantation is required when the glomerular filtration rate falls below 10% to 15% of normal. Anemia and acidosis may be present as part of the underlying disorder. The glomerular filtration rate determines the need for dialysis. The kidneys are able to maintain the chemical composition of fluids within normal limits until more than 50% of functional renal capacity is destroyed by disease or injury.

113. A child has been diagnosed with hepatitis A and received treatment. Based on this information the nurse determines that the illness was transmitted via blood route. immunity has been acquired for this type. crossover immunity is present for all types of hepatitis. the patient will now be a carrier for this type.

immunity has been acquired for this type. Once a patient has been exposed and treated, they develop immunity to this type but there is no crossover immunity to other hepatitis types. Hepatitis A if transmitted through fecal-oral route and is not blood borne. There is no carrier state for Hepatitis A.

77. A young child is brought to the emergency department with severe dehydration secondary to acute diarrhea and vomiting. Therapeutic management of this child begins with intravenous fluids. oral rehydration solution. clear liquids, 1 to 2 oz at a time. administration of antidiarrheal medication.

intravenous fluids. In children with severe dehydration, intravenous fluids are initiated. Oral rehydration solution is acceptable therapy if the dehydration is not severe. Diarrhea is not managed by using clear liquids by mouth. These fluids usually have a high carbohydrate content, low electrolyte content, and high osmolality. Antidiarrheal medications are not recommended for the treatment of acute infectious diarrhea.

45. The callus that develops at the fracture site is important because it provides: functional use of injured part. sufficient support for weight bearing. means for adequate blood supply. means for holding bone fragments together.

means for holding bone fragments together. Functional use cannot occur until the fracture site is stable. Functional use cannot occur until the fracture site is stable. The callus does not provide an adequate blood supply. New bone cells are formed in large numbers and are stimulated to maximum activity. They are found at the site of the injury. In time, calcium salts are absorbed to form the callus.

64. An important nursing intervention when caring for a child with myelomeningocele in the postoperative stage is to: place child on his or her side to decrease pressure on the spinal cord. apply a heat lamp to facilitate drying and toughening of the sac. keep skin clean and dry to prevent irritation from diarrheal stools. measure head circumference and examine fontanels for signs that might indicate developing hydrocephalus.

measure head circumference and examine fontanels for signs that might indicate developing hydrocephalus. Before surgery the child is kept in a prone position to decrease tension on the sac and reduce risk of trauma. The sac must be kept moist. Sterile, moist, nonadherent dressings are placed over the sac. Most infants do not have diarrheal stools. Hydrocephalus is frequently associated with myelomeningocele. Assessment of the fontanels and daily measurements of head circumference will aid in early detection.

55. A 15-year-old is admitted to the intensive care unit (ICU) with a spinal cord injury. The MOST appropriate nursing interventions for this adolescent are: (Select all that apply.) monitoring neurologic status. administering corticosteroids. monitoring for respiratory complications. discussing long-term care issues with the family. monitoring and maintaining hemodynamic status.

monitoring neurologic status. administering corticosteroids. monitoring for respiratory complications. monitoring and maintaining hemodynamic status. Close monitoring of sensory and motor function is important to prevent further deterioration of neurologic status as a result of spinal cord edema. Corticosteroids are administered to minimize the inflammation associated with the injury. Close monitoring of respiratory status for possible need of ventilator support. Remember "A-B-C's," airway, breathing, and circulation. Monitoring and maintaining hemodynamic status may require immediate attention related to increased intracranial pressure resulting in hypotension and bradycardia. The discussion of long-term care issues with the family is not appropriate in the acute phase of spinal cord injury.

52. An 8-year-old child is hospitalized with infectious polyneuritis (Guillain-Barré syndrome). When explaining this disease process to the parents, the nurse should consider that: paralysis is progressive with little hope for recovery. muscle function will gradually return, and recovery is possible in most children. disease results from an apparently toxic reaction to certain medications. disease is inherited as an autosomal, sex-linked, recessive gene.

muscle function will gradually return, and recovery is possible in most children. The paralysis is progressive, but most children have full recovery. Supportive nursing care is essential. Most patients regain full muscle strength. The return of function is in reverse order of onset. It is an immune-mediated disease associated with viral and bacterial infections. It is an immune-mediated disease associated with viral and bacterial infections.

130. A 10-year-old with glomerulonephritis reports a headache and blurred vision. The nurse should immediately:

obtain the child's blood pressure. Rationale: Hypertension occurs with acute glomerulonephritis. The symptoms of headache and blurred vision may indicate an elevated blood pressure. Hypertension in acute glomerulonephritis occurs due to the inability of the kidneys to remove fluid and sodium; the fluid is reabsorbed, causing fluid volume excess. The nurse must verify that these symptoms are due to hypertension.

82. A nurse working in triage in the emergency room is assessing a pediatric patient, age 4, who presents with pain in the wrist. The patient refuses to move the involved extremity. Questioning of the patient and parent reveal no trauma event. Based on this observation the nurse suspects that the patient may have compound fracture. partial dislocation or subluxation. complication of immobilization. overuse of extremity.

partial dislocation or subluxation. A common occurrence seen in young children is subluxation, partial dislocation of the radial head, called pulled elbow or nursemaid's elbow. This occurs in response to a pulling traction response due to an individual pulling the child's extremity in response to an action to protect the child. It is not seen as a complication of immobilization or overuse of the extremity. The clinical presentation is not consistent with a compound fracture as the bone would be protruding from the skin.

140. The mother of a child with tetralogy of Fallot asks the nurse why her child has clubbed fingers. The nurse bases the response on the understanding that clubbing is due to which factor?

peripheral hypoxia Rationale: Clubbing of the fingers is one common finding in the child with persistent hypoxia, which leads to tissue changes in the body because of the low oxygen content of the blood (hypoxemia). It apparently results from tissue fibrosis and hypertrophy from the hypoxemia and from an increase in capillaries in the area, which occur as the body attempts to improve the blood supply

26. A 5-year-old female child has been sent to the school nurse for urinary incontinence 3 times in the past 2 days. The nurse should recommend to her parent that the FIRST action is to have the child evaluated for: 1. school phobia. 2. emotional causes. 3. possible urinary tract infection. 4. possible structural defects of the urinary tract.

possible urinary tract infection. A physical cause of the problem needs to be eliminated before a psychologic cause is considered. A physical cause of the problem needs to be eliminated before a psychologic cause is considered. Incontinence in a previously toilet trained child can be an indication of a urinary tract infection. Structural defects would be explored after a urinary tract infection is confirmed.

49. Cerebral palsy may result from a variety of causes. It is now known that the most common cause of cerebral palsy is: birth asphyxia. neonatal diseases. cerebral trauma. prenatal brain abnormalities.

prenatal brain abnormalities. These issues were previously thought to be factors. These issues were previously thought to be factors. These issues were previously thought to be factors. Cerebral palsy results from existing brain abnormalities during the prenatal period.

80. Enteral feedings are ordered for a young child with burns covering 40% of total body surface area. The nurse should know that oral feedings are contraindicated. enteral feedings must be stopped during painful procedures. presence of a paralytic ileus does not preclude use of enteral feedings. the feedings will be high carbohydrate and low protein.

presence of a paralytic ileus does not preclude use of enteral feedings. Because the small bowel maintains mobility and absorptive capabilities, the placement of a small-bore feeding tube into the duodenum allows for safe delivery of enteral nutrition during periods of paralytic ileus associated with sepsis, trauma, and anesthesia. Oral feedings are not contraindicated. This is encouraged; however, most children with burns are unable to consume sufficient calories by mouth. Enteral feedings can continue during procedures. A high-protein, high-calorie diet is recommended.

37. Major goals of the therapeutic management of juvenile rheumatoid arthritis are to: prevent joint discomfort and regain proper alignment. prevent loss of joint function and achieve cure. prevent physical deformity and preserve joint function. prevent skin breakdown and relieve symptoms.

prevent physical deformity and preserve joint function. Once the joint is damaged, it may not be possible to regain proper alignment. It may not be possible to achieve a cure. These are the goals of treatment. A third goal is to control pain. Skin breakdown is usually not an issue in juvenile rheumatoid arthritis.

27. External defects of the genitourinary tract such as hypospadias are usually repaired as early as possible to: 1. prevent urinary complications. 2. prevent separation anxiety. 3. promote acceptance of hospitalization. 4. promote development of normal body image.

promote development of normal body image. Preventing urinary complications is important for defects that affect function, but all external defects should be repaired as soon as possible. Proper preprocedure preparation can facilitate coping with these issues. Proper preprocedure preparation can facilitate coping with these issues. This is extremely important. Surgery involving sexual organs can be very upsetting to children, especially preschoolers who fear mutilation and castration.

50. A child with spina bifida has developed a latex allergy from numerous bladder catheterizations and surgeries. A PRIORITY nursing intervention is to: recommend allergy testing. provide a latex-free environment. use only powder-free latex gloves. limit use of latex products as much as possible.

provide a latex-free environment. This may expose the child to the allergen; it is not recommended. This is the most important nursing intervention. From birth on, the limitation of exposure to latex is essential in an attempt to minimize sensitization. The gloves contain latex and will contribute to sensitization. Latex products should be avoided.

131. A 4-year-old child is having a sickle cell crisis. The initial nursing intervention should be to:

provide oral and I.V. fluids. Rationale: Initial nursing interventions for the child in a sickle cell crisis include providing hydration and oxygenation to prevent more sickling. Pain relief is also a concern. However, painful joints are treated with analgesics and warm packs because cold packs may increase sickling.

146. Which finding would indicate that an infant with a tracheoesophageal fistula (TEF) needs suctioning?

substernal retractions Rationale: With a TEF, overflow of secretions into the larynx leads to laryngospasm. This obstruction to inspiration stimulates the strong contraction of accessory muscles of the thorax to assist the diaphragm in breathing. This produces substernal retractions. The laryngospasm that occurs with a TEF resolves quickly when secretions are removed from the oropharynx area.

47. An adolescent who had a lower leg amputated after a motorcycle accident complains of pain in the missing extremity. The nurse's MOST appropriate action is to: withhold pain medications because narcotic addiction. refer the patient for psychological counseling. teach the parents and adolescent child about nerve damage. reassure the child that it is normal and is called phantom limb sensation.

reassure the child that it is normal and is called phantom limb sensation. Phantom limb sensation is an expected phenomena following amputation of an extremity. The other choices are not relevant. Phantom limb sensation is an expected phenomena following amputation of an extremity. The other choices are not relevant. Phantom limb sensation is an expected phenomena following amputation of an extremity. The other choices are not relevant. Phantom limb sensation is an expected experience because the nerve-brain connections are still present. They gradually fade. This should be discussed before surgery with the child.

57. The major goals of therapy for children with cerebral palsy include: reversing degenerative processes that have occurred. curing underlying defect causing the disorder. preventing spread to individuals in close contact with the child. recognizing the disorder early and promoting optimal development.

recognizing the disorder early and promoting optimal development. It is very difficult to reverse degenerative processes. The underlying defect cannot be cured. Cerebral palsy is not contagious. Since cerebral palsy is currently a permanent disorder, the goal of therapy is to promote optimal development. This is done through early recognition and beginning of therapy.

25. A young child is diagnosed with vesicoureteral reflux. The nurse should know that this usually is associated with: 1. incontinence. 2. urinary obstruction. 3. recurrent kidney infections. 4. infarction of renal vessels.

recurrent kidney infections. Incontinence may be associated with urinary tract infections. When reflux is associated with vesicoureteral reflux, it can cause renal scarring but not obstruction. Reflux allows urine to flow back to the kidneys. When the urine is infected, this contributes to kidney infections. Infarction of renal vessels does not occur.

60. A neural tube defect that is not visible externally in the lumbosacral area would be called: meningocele. myelomeningocele. spina bifida cystica. spina bifida occulta.

spina bifida occulta. Meningocele contains meninges and spinal fluid but no neural tissue. Unless there are associated cutaneous findings, it is often not identified until later. Myelomeningocele is a neural tube defect that contains meninges, spinal fluid, and nerves. This is a cystic formation with an external saclike protrusion. Spina bifida occulta is completely enclosed. Often this defect will not be noticed.

58. A 3-year-old male child has cerebral palsy and is currently hospitalized for orthopedic surgery. His mother says that he has difficulty swallowing and cannot hold a utensil to feed himself. He is slightly underweight for his height. The MOST appropriate nursing action related to feeding this child is to: bottle- or tube-feed him a specialized formula until he gains sufficient weight. stabilize his jaw with one hand (either from a front or side position) to facilitate swallowing. place him in a well-supported, semireclining position to make use of gravity flow. place him in a sitting position with his neck hyperextended to make use of gravity flow.

stabilize his jaw with one hand (either from a front or side position) to facilitate swallowing. Age 3 is too old for bottle-feeding. The neuromuscular compromise of the jaw interferes with the child's ability to eat. Because the jaw is compromised, more normal control can be achieved if the feeder provides stability. Manual jaw control assists with head control, correction of neck and trunk hyperextension, and jaw stabilization. The child should be sitting up for meals. For swallowing, the neck should not be hyperextended.

42. The nurse is caring for an immobilized preschool child. During this period of immobilization, the nurse's BEST action is to: encourage wearing pajamas. let the child have few behavioral limitations. keep child away from other immobilized children if possible. take child for a "walk" by wagon outside the room.

take child for a "walk" by wagon outside the room. The child should be encouraged to wear street clothes during the daytime. Limit setting is necessary with all children. There is no reason to segregate children who are immobilized unless there are other medical issues that need to be addressed. It is important for children to have activities outside of the room if possible. This increases environmental stimuli and provides social contact with others.

62. A 6-year-old girl born with a myelomeningocele has a neurogenic bladder disorder. Her parents have been performing clean intermittent catheterization. The nurse's MOST appropriate action is to: teach the child to do self-catheterization. teach the child appropriate bladder control. continue having parents do catheterization. encourage the family to consider urinary diversion.

teach the child to do self-catheterization. At 6 years old this child should be able to perform the intermittent catheterization herself. This will give her more control and mastery over her disability. Bladder control cannot be taught to a child with a neurogenic bladder. This would be a good time to have the child begin caring for herself. A urinary diversion is not necessary.

29. In a non-potty-trained child with nephrotic syndrome, the best way to detect fluid retention is to: 1. weigh the child daily. 2. test the urine for hematuria. 3. measure the abdominal girth weekly. 4. count the number of wet diapers.

weigh the child daily. Measuring weight at the same time each day is the most accurate way to determine fluid gains and losses. The presence or absence of blood in the urine will not help with the determination of fluid retention. Abdominal girth is reflective of edema, but weekly is too infrequent a measure. The number of wet diapers reflects how often they have been changed. The diapers should be weighed to reflect fluid balance.

139. During physical assessment of a 4-month-old infant with Hirschsprung's disease, the nurse should most likely note which finding?

weight less than expected for height and age Rationale: Infants with Hirschsprung's disease typically display failure to thrive, with poor weight gain due to malabsorption of nutrients. Would have a distended abdomen


संबंधित स्टडी सेट्स

RN Maternal Newborn Online Practice 2023 A

View Set

Spanish III conditional/future oral

View Set

Chapter 7 Interest Rates and Bond Valuation (Quiz)

View Set

EOSC 114 Final Exam (Learning Goals)

View Set